Download as pdf or txt
Download as pdf or txt
You are on page 1of 179

STUDY SESSION 1: THE NUCLEAR ATOM

INTRODUCTION
Previously, you have learnt about the basics of the electron in Modern Physics I (PHS 204). We
shall take a step further in explaining its behavior in an atom. Typically, an atom is made up of a
small nucleus of protons and neutrons with a number of electrons located at some distance away
from the nucleus.

LEARNING OUTCOMES:
At the end of the lesson, the student should be able to;
1.1 Explain briefly development of the atomic structure
1.2 State the expression for electron velocity and the energy of a hydrogen atom
1.3 State the Rutherford’s formula
1.4 Highlight the failures of the classical theory

1.1 NUCLEAR ATOM


Most scientists of the late nineteenth century accepted the idea that the chemical elements consist
of atoms, but they knew almost nothing about the atoms themselves. One clue was the discovery
that all atoms contain electrons. Since electrons carry negative charges, positively charged matter
of some kind must be present in atoms for atoms to remain electrically neutral.

In 1897, the English physicist J. J. Thomson (Nobel Prize 1906) had discovered the electron and
measured its charge-to-mass ratio 𝑒⁄𝑚. The best available model of atomic structure was one
developed by Thomson at that time. He envisioned the atom as a sphere of some as yet
unidentified positively charged substance, within which the electrons were embedded like raisins
in cake. In an African sense, one could picture a Thompson atom much like a spherically
symmetric guava fruit. This model offered an explanation for line spectra. If the atom collided
with another atom, as in a heated gas, each electron would oscillate around its equilibrium
1
position with a characteristic frequency and emit electromagnetic radiation with that frequency.
If the atom were illuminated with light of many frequencies, each electron would selectively
absorb only light whose frequency matched the electron’s natural oscillation frequency.

By 1909, the American physicist Robert Millikan (Nobel Prize 1923) had made the first
measurements of the electron charge – 𝑒. This and other experiments showed that almost all the
mass of an atom had to be associated with the positive charge, not with the electrons. It was also
known that the overall size of atoms is of the order of 10−10and that all atoms except hydrogen
contain more than one electron.

The experiments of Geiger and Marsden in 1911 and later work of a similar kind also supplied
information about the nuclei of the atoms that composed the various target foils. At the
suggestion of Ernest Rutherford, they used as probes the fast alpha particles emitted by certain
radioactive elements. Alpha particles are helium atoms that have lost two electrons each, leaving
them with a charge of +2e. Geiger and Marsden placed a sample of an alpha-emitting substance
behind a lead screen with a small hole in it, as in Figure 1.1, so that a narrow beam of alpha
particles was produced. This beam was directed at a thin gold foil.

A zinc sulfide screen, which gives off a visible flash of light when struck by an alpha particle,
was set on the other side of the foil with a microscope to see the flashes. It was expected that the
alpha particles would go right through the foil with hardly any deflection. This follows from the
Thomson model, in which the electric charge inside an atom is assumed to be uniformly spread
through its volume. With only weak electric forces exerted on them, alpha particles that pass
through a thin foil ought to be deflected only slightly, 1° or less.

2
Figure 1.1: Rutherford scattering experiment

To their amazement, Geiger and Marsden actually found that although most of the alpha particles
indeed were not deviated much, a few were scattered through very large angles. Some were even
scattered in the backward direction. As Rutherford remarked, “It was as incredible as if you fired
a 15-inch shell at a piece of tissue paper and it came back and hit you.” Alpha particles are
relatively heavy (almost 8000 electron masses) and those used in this experiment had high
speeds (typically 2 × 107 m/s), so it was clear that powerful forces were needed to cause such
marked deflections.
3
The deflection of an alpha particle when it passes near a nucleus depends on the magnitude of
the nuclear charge. Comparing the relative scattering of alpha particles by different foils thus
provides a way to find the nuclear charges of the atoms involved. The nuclear charges always
turned out to be multiples of +e. The number Z of charges in the nuclei of an element is called
the atomic number of the element. Thus we can say that the atomic number of an element is the
number of protons in the nuclei of the atom.

Later experiments showed that all nuclei are composed of positively charged protons (discovered
in 1918) and neutrons that are electrically neutral (discovered in 1930). For example, the gold
atoms in Rutherford’s experiments have 79 protons and 118 neutrons. In fact, an alpha particle is
itself the nucleus of a helium atom, with two protons and two neutrons. It is much more massive
than an electron but only about 2% as massive as a gold nucleus, which helps explain why alpha
particles are scattered by gold nuclei but not by electrons.

However, According to Maxwell’s electromagnetic theory, a charged particle in circular motion


radiates energy and so an electron in a Rutherford’s atom should continuously lose energy as it
moves in a planetary orbit and eventually should spiral into to the nucleus at the centre of the
atom, which does not happen. Rutherford’s model, though a much improved picture of the atom,
but could not explain stability of the atom.

Furthermore, according to classical physics, the energy emitted by an electron as it spirals into
the nucleus should have all frequencies. In other words the emitted spectrum should be
continuous which is not the case. The emitted spectrum consist of lines in a dark background.
Thus, Rutherford’s model could not explain the observed line spectra of elements.

4
1.2 ELECTRON ORBIT
We shall now consider the classical dynamics of the hydrogen atom. The hydrogen atoms is the
simplest of all atoms because of its single electron. We assume a circular electron orbit for
convenience as shown in Figure 1.2.

Figure 1.2: Force balance in the hydrogen atom

It follows that from the Rutherford model, the atom consists of a massive, positively charged
nucleus surrounded at a relatively great distance by enough electrons to make the atom
electrically neutral as a whole. The electrons cannot be stationary in this model, because there is
nothing that can keep them in place against the electric force pulling them to the nucleus.

Hence, the centripetal force of the electron is given as


𝑚𝑣 2
𝐹𝑐 = 1.1
𝑟

Also, the electrical force between the nucleus and the electron is given as

𝑒2
𝐹𝑒 = 4𝜋𝜖 2
1.2
𝑜𝑟

5
The centripetal force holding the electron in an orbit r from the nucleus is provided by this
electric force.
Thus, the condition for balancing of the forces is
𝐹𝑐 = 𝐹𝑒 1.3
𝑚𝑣 2 𝑒2
= 1.4
𝑟 4𝜋𝜖𝑜 𝑟 2

The electron velocity v is therefore related to its orbit radius r by the formula
𝑒
𝑣= 1.5
√4𝜋𝜖𝑜 𝑚𝑟

Where e = electronic charge,


m = mass of electron
𝜖𝑜 = permittivity of free space
The total energy E of the electron in a hydrogen atom is the sum of its kinetic and potential
energies, which are
1 −𝑒 2
𝐾. 𝐸 = 𝑚𝑣 2 𝑎𝑛𝑑 𝑃. 𝐸 = 1.6
2 4𝜋𝜖𝑜 𝑟

The minus sign follows from the choice of PE = 0 at 𝑟 = ∞ that is, when the electron and proton
are infinitely far apart.
Hence, total energy is
1 𝑒2
𝐸 = 𝐾. 𝐸 + 𝑃. 𝐸 = 𝑚𝑣 2 − 1.7
2 4𝜋𝜖𝑜 𝑟

Substituting 1.5 into 1.7, we have


𝑒2 𝑒2
𝐸= −
8𝜋𝜖𝑜 𝑟 4𝜋𝜖𝑜 𝑟

𝑒2
𝐸=− 1.8
8𝜋𝜖𝑜 𝑟

Equation 1.8 gives the total energy of the hydrogen atom

6
The total energy of the electron is negative. This holds for every atomic electron and reflects the
fact that it is bound to the nucleus. If E were greater than zero, an electron would not follow a
closed orbit around the nucleus. Actually, of course, the energy E is not a property of the electron
alone but is a property of the system of electron and nucleus.

Experiments indicate that 13.6 eV is required to separate a hydrogen atom into a proton
and an electron; that is, its total energy is E = -13.6 eV. Find the orbital radius and velocity
of the electron in a hydrogen atom.

From equation 1.8, E = 13.6eV = 13.6eV x 1.6 × 10−19 C = 2.2 x 10-18J


𝑒2 (1.6 × 10−19 )2
𝑟= − = −
8𝜋𝜖𝑜 𝐸 8𝜋(8.85 × 10−12 )(−2.2 × 10−18 )
= 5.3 × 10−11 𝑚
An atomic radius of this magnitude agrees with estimates made in other ways. The electron’s
velocity can be found from equation 1.5
𝑒 1.6 × 10−19
𝑣= =
√4𝜋𝜖𝑜 𝑚𝑟 √4𝜋(8.85 × 10−12 )(9.1 × 10−31 )(5.3 × 10−11 )
= 2.2 × 106 𝑚/𝑠
Since 𝑣 ≪ 𝑐, we can ignore special relativity when considering the hydrogen atom.

1.3 RUTHERFORD SCATTERING

7
Rutherford scattering is a phenomenon that was expounded by Ernest Rutherford in 1911. His
explanation provided the basis for the development of the orbital theory of the atom. It is now
exploited by the materials analytical technique Rutherford backscattering. This scattering is
sometimes referred to as Coulomb scattering because it relies on static electric forces.

Rutherford concluded that the larger percentage of the mass was concentrated the nucleus. The
nucleus is surrounded by electrons. When a (positive) alpha particle approached sufficiently
close to the nucleus, it was repelled strongly enough to rebound at high angles. The small size of
the nucleus explained the small number of alpha particles that were repelled in this way.
Rutherford showed, using the method below, that the size of the nucleus was less than about
10−14 m .

Scattering Theory
The main assumptions of this theory are as follows

• The collision between a point charge but heavy nucleus with charge Q=Ze and a light
projectile with charge q = ze is considered to be elastic,
• The particles interact by the Coulomb force;
• The energy and momentum are conserved,
• The vertical distance the projectile is from the centre of the target, the impact parameter
b, determines the scattering angle .

8
Figure 1.3 Geometry of Scattering

1
The relationship between the scattering angle, , the initial kinetic energy 𝐾 = 𝑚𝑣𝑜2 and the
2

impact parameter b is given by

𝑧𝑍 𝑒2 𝜃
𝑏= 𝑐𝑜𝑡 2 1.9
2𝐾 4𝜋𝜀𝑜

where Z =2 for  -particle, Z = 79 for gold, e = 1.6 × 10−19 C. k = 8.98 × 109 𝜇𝑚2 𝑐 −2

Find the fraction of a beam of 7.7MeV alpha particles that is scattered through angles of more
than 45° when incident upon a gold foil 3 x 10-7m thick. These values are typical of the alpha
particle energies and foil thicknesses used by Geiger and Marsden. For comparison, a human hair
is about 10-4 m in diameter.

9
We begin by finding n, the number of gold atoms per unit volume in the foil, from the
relationship
𝑎𝑡𝑜𝑚𝑠 𝑚𝑎𝑠𝑠/𝑚3
𝑛= =
𝑚3 𝑚𝑎𝑠𝑠/𝑎𝑡𝑜𝑚
Since the density of gold is 1.93 x 104 kg/m3, its atomic mass is 197 u, and 1 u = 1.66 x10-27 kg,
we have
1.93 × 10−4
𝑛=
(197)(1.66 × 10−27 )

n = 5.90 x 1028 atoms/m3


The atomic number Z of gold is 79, a kinetic energy of 7.7 MeV is equal to 1.23 x 10-12 J, and Ө
= 45°; from these figures we find that
f = 7 x 10-5
of the incident alpha particles are scattered through 45°. A foil this thin is quite transparent to
alpha particles.

A Cursory Derivation of the Differential Cross section

In Figures 1.3 and 1.4, a particle that hits the ring between b and b + db is scattered into the solid
angle d between  and  + d.

By definition, the cross section is the proportionality constant

2𝜋𝑏𝑑𝑏 = − 𝜎(𝜃)2𝜋𝑠𝑖𝑛𝜃𝑑𝜃

Hence,

𝑑𝜎
𝑑𝜎 = 2𝜋𝑏|𝑑𝑏| = ( ) 𝑑Ω 1.10
𝑑Ω
10
where 𝑑Ω = 2𝜋𝑠𝑖𝑛𝜃𝑑𝜃

The Differential Cross Section then becomes

𝑑𝜎 2𝜋𝑏|𝑑𝑏|
= 1.11
𝑑Ω 2𝜋𝑠𝑖𝑛𝜃𝑑𝜃

From Equations 1.9 and 1.11 we have

𝑑𝜎 1 2 𝑞𝑄 2 1
= ( ) (4𝐾 ) 𝜃 1.12
𝑑Ω 4𝜋𝜀 𝑜 𝛼 𝑠𝑖𝑛4 ( )
2

Equation 1.12, is called the Differential Cross section for Rutherford Scattering.

Figure 1.4 Schematic Geometry for Calculation of Scattering Cross Section

11
Figure 1.5 Detailed Geometrical Arrangements for Calculation of Scattering Cross
Section

In the above calculations, only a single  - particle is considered. In a scattering experiment, one
must consider multiple scattering events and one measures the fraction of particles scattered
through a given angle.

For a detector at a specific angle with respect to the incident beam, the number of particles per
unit area striking the detector is given by the Rutherford formula:

𝑁𝑖 𝑛𝐿𝑍 2 𝑘 2 𝑒 4
𝑁(𝜃 ) = 𝜃 1.13
4𝑟 2 𝐾𝐸 2 𝑠𝑖𝑛2 ( )
2

Where Ni = number of incident  - particles,


n = atoms per unit volume in target
L = thickness of the target
Z = atomic number of target
e = electronic charge
k = Coulomb’s constant

12
r = target to detector distance,
KE = kinetic energy of  - particles and
 = scattering angle.

The predicted variation of detected alphas with angle is followed closely by the Geiger-Marsden
data, shown in Fig below.

Figure 1.6: Verification of Rutherford’s Formula

Calculation of Maximal Nuclear Size

For head on collisions between alpha particles and the nucleus, all the kinetic energy of the alpha
particle is turned into potential energy and the particle is at rest. The distance from the center of
the alpha particle to the center of the nucleus (b) at this point is a maximum value for the radius,
if it is evident from the experiment that the particles have not hit the nucleus.
13
Figure 1.7 Scattering with Different Impact Parameters

Applying the Coulomb potential energy between the charges on the electron and nucleus, one
1 1 𝑞1 𝑞2
can write: 𝑚𝑣 2 =
2 4𝜋𝜀𝑜 𝑏

Rearranging:

1 2𝑞1 𝑞2
𝑏= 1.14
4𝜋𝜀𝑜 𝑚𝑣 2

For an alpha particle:

• m (mass) = 6.7×10−27 kg

• 𝑞1 = 2×(1.6×10−19) C

• 𝑞2 (for gold) = 79×(1.6×10−19) C

• v (initial velocity) = 2×107 m/s

Substituting these parameters into Equation 1.14, gives the value of the impact parameter of
about 2.7×10−14m. The true radius is about 7.3×10−15 m.

14
1.4 FAILURE OF CLASSICAL PHYSICS
Rutherford’s discovery of the atomic nucleus raised a serious question: What prevented the
negatively charged electrons from falling into the positively charged nucleus due to the strong
electrostatic attraction? Rutherford suggested that perhaps the electrons revolve in orbits about
the nucleus, just as the planets revolve around the sun.
But according to classical electromagnetic theory, any accelerating electric charge (either
oscillating or revolving) radiates electromagnetic waves. An example is the radiation from an
oscillating point charge. An electron orbiting inside an atom would always have a centripetal
acceleration toward the nucleus, and so should be emitting radiation at all times. The energy of
an orbiting electron should therefore decrease continuously, its orbit should become smaller and
smaller, and it should spiral into the nucleus within a fraction of a second. Even worse, according
to classical theory the frequency of the electromagnetic waves emitted should equal the
frequency of revolution. As the electrons radiated energy, their angular speeds would change
continuously, and they would emit a continuous spectrum (a mixture of all frequencies), not the
line spectrum actually observed. Thus Rutherford’s model of electrons orbiting the nucleus,
which is based on Newtonian mechanics and classical electromagnetic theory, makes three
entirely wrong predictions about atoms: They should emit light continuously, they should be
unstable, and the light they emit should have a continuous spectrum.

SUMMARY
𝑒
Electron velocity is given as 𝑣 =
√4𝜋𝜖𝑜 𝑚𝑟

𝑒2
Total energy of the hydrogen atom is given as 𝐸 = −
8𝜋𝜖𝑜 𝑟

Rutherford formula:

15
𝑁𝑖 𝑛𝐿𝑍 2 𝑘 2 𝑒 4
𝑁(𝜃 ) = 𝜃
4𝑟 2 𝐾𝐸 2 𝑠𝑖𝑛2 ( 2 )

The failure of classical physics are:


➢ Atoms should emit light continuously but could not explain the photoelectric emission.
➢ Atoms should be unstable
➢ Light emitted by atoms should have a continuous spectrum i.e. could not explain the
phenomenon of line spectra.

SELF ASSESSMENT QUESTIONS


(1) Almost the entire mass of an atom is concentrated in the__________.
A. proton
B. electrons
C. nucleus
D. neutrons
(2) The fixed circular paths around the nucleus are called_________.
A. orbits
B. orbitals
C. nucleons
D. mesons
(3) Electron was discovered by___________.
A. Chadwick
B. Thomson
C. Goldstein
D. Bohr
(4) The volume of the nucleus of an atom when compared to the extra nuclear part is_________.
A. bigger
B. smaller

16
C. same size
D. unpredictable
(5) The absolute charge of an electron is___________.
A. - 1.6 x 10-19C
B. + 1.6 x 10-19C
C. 1.6 x 10-19C
D. 16 x 10-19C

(6) Which model of atomic structure


was developed to explain the results of the
experiment shown?
alpha metal screen
(a) Bohr model
particle foil
(b) quantum mechanical atom
source
(c) plum-pudding model
(d) nuclear atom

(7) In the planetary model of the atom where electrons orbit a centralized nucleus, what
is the approximate ratio of the radius of the nucleus to that of the electron orbits,
rn/re?
(a) 105
(b) 103
(c) 10−3
(d) 10−5

(8) A beam of 8.0-MeV α particles scatters from a thin gold foil. What is the ratio of the
number of α particles scattered to angles greater than 1° to the number scattered to angles
greater than 2°?

17
(10) For aluminium (Z = 13) and gold (Z = 79) targets, what is the ratio of an alpha particle
scattering at any angle for equal numbers of scattering nuclei per unit area?

(10) Derive the Rutherford scattering formula

REFERENCES
Hugh D. Young and Roger A. Freedman, (2004), University Physics with Modern Physics,11 th
Edition, Pearson Education Publishers

Arthur Beiser (2003), Concepts of Modern Physics, 6th Edition, McGraw-Hill Publishers

Kiwanga C.A., Atomic Physics module 8, The Open University of Tanzania

Should you require more explanation on this study session, please do not hesitate to contact your

e-tutor via the LMS.

Are you in need of General Help as regards your studies? Do not hesitate to contact
the DLI IAG Center by e-mail or phone on:

iag@dli.unilag.edu.ng
08033366677

18
STUDY SESSION 2: ATOMIC SPECTRA

INTRODUCTION
Every neutral atom contains at least one electron. As we will see, it is crucial to understand not
only the structure of atoms but also how they interact with light. Historically, the quest to
understand the nature of the atom was intimately linked with both the idea that electrons have
wave characteristics and the notion that light has particle characteristics. Atomic stability is not
the only thing that a successful theory of the atom must account for. The existence of spectral
lines is another important aspect of the atom that finds no explanation in classical physics.

LEARNING OUTCOMES
At the end of this study session, the students should be able to:
2.1 Explain briefly the formation of line spectra
2.2 Itemize the spectral series
2.3 State the Bohr model of an atom
2.4 State the expression to determine the energy levels of the hydrogen atom

2.1 LINE SPECTRA

In the world of physics, we know that heated materials emit light, and that different materials
emit different kinds of light. The coils of a toaster glow red when in operation and the flame of a
match has a characteristic yellow color. To analyze these different types of light, we can use a
prism or a diffraction grating to separate the various wavelengths in a beam of light into a
spectrum. If the light source is a hot solid (such as the filament of an incandescent light bulb) or
liquid, the spectrum is continuous; light of all wavelengths is present.

When an atomic gas or vapor such as the neon in an advertising sign or the sodium vapor formed
when table salt is thrown into a campfire, at somewhat less than atmospheric pressure is suitably
19
“excited,” usually by passing an electric current through it, the emitted radiation has a spectrum
which contains certain specific wavelengths only. An idealized arrangement for observing such
atomic spectra is shown in Figure 2.1.

Figure 2.1 An idealized spectrometer

Actual spectrometers use diffraction gratings. Figure 2.2 shows the emission line spectra of
several elements and the lines are called spectral lines. Each spectral line corresponds to a
definite wavelength and frequency.

20
Figure 2.2 Some of the principal lines in the emission spectra of hydrogen, helium, and
mercury.

Every element displays a unique line spectrum when a sample of it in the vapor phase is excited.
Spectroscopy is therefore a useful tool for analyzing the composition of an unknown substance.
When white light is passed through a gas, the gas is found to absorb light of certain of the
wavelengths present in its emission spectrum. The resulting absorption line spectrum consists
of a bright background crossed by dark lines that correspond to the missing wavelengths.
Emission spectra consist of bright lines on a dark background.

The spectrum of sunlight has dark lines in it because the luminous part of the sun, which radiates
very nearly like a blackbody heated to 5800 K, is surrounded by an envelope of cooler gas that
absorbs light of certain wavelengths only. Most other stars have spectra of this kind. The
number, intensity, and exact wavelengths of the lines in the spectrum of an element depend upon
temperature, pressure, the presence of electric and magnetic fields, and the motion of the source.
E. coli bacteria
2.2 SPECTRAL SERIES
Over a century ago the wavelengths in the spectrum of an element were found to fall into sets
called spectral series. The first such series was discovered by Johann J. Balmer in 1885 in the
course of a study of the visible part of the hydrogen spectrum. Figure 2.3 shows the Balmer
series.

21
Figure 2.3 Spectral series of hydrogen with Balmer series between 365nm and 656nm.

The line with the longest wavelength, 656.3 nm, is designated Hα, the next, whose wavelength is
486.3 nm, is designated Hβ, and so on. As the wave-length decreases, the lines are found closer
together and weaker in intensity until the series limit at 364.6 nm is reached, beyond which there
are no further separate lines but only a faint continuous spectrum. Balmer’s formula for the
wavelengths of this series is

2.1

The quantity R, known as the Rydberg constant, has the value R = 1.097 x 107 m-1 = 0.01097
nm-1.

Find
(a) The longest and
(b) The shortest wavelengths of the Balmer series.

Each wavelength in the series corresponds to one value for the integer n in equation 2.1. Longer
wavelengths are associated with smaller values of n. The longest wavelength occurs when n has
its smallest value of n = 3. The shortest wavelength arises when n has a very large value, so that
1/n2 is essentially zero.

22
a With n = 3, Equation 2.1 reveals that for the longest wavelength

1 1 1 1 1
= 𝑅 ( 2 − 2 ) = (1.097 × 107 𝑚−1 ) ( 2 − 2 ) = 1.524 × 106 𝑚−1 𝑜𝑟
𝜆 2 𝑛 2 3
𝜆 = 656𝑛𝑚
b. With 1/n2 = 0, Equation 2.1 reveals that for the shortest wavelength
1 1
= (1.097 × 107 𝑚−1 ) ( 2 − 0 ) = 2.743 × 106 𝑚−1 𝑜𝑟 𝜆 = 365𝑛𝑚
𝜆 2

The Hα line corresponds to n = 3, the Hβ line to n = 4, and so on. The series limit corresponds to
𝑛 = ∞, so that it occurs at a wavelength of 4/R, in agreement with experiment.

The Balmer series contains wavelengths in the visible portion of the hydrogen spectrum. The
spectral lines of hydrogen in the ultraviolet and infrared regions fall into several other series. In
the ultraviolet the Lyman series contains the wavelengths given by the formula.

1 1 1
Lyman = = 𝑅( 2− ) 𝑛 = 2,3,4 … … 2.2
𝜆 1 𝑛2

In the infrared, three spectral series have been found whose lines have the wavelengths specified
by the formulas

1 1 1
Paschen = = 𝑅( − ) 𝑛 = 4,5,6 … … 2.3
𝜆 32 𝑛2
1 1 1
Brackett = = 𝑅( 2 − ) 𝑛 = 5,6,7 … … 2.4
𝜆 4 𝑛2
1 1 1
Pfund = = 𝑅( − ) 𝑛 = 6,7,8 … … 2.5
𝜆 52 𝑛2

23
These spectral series of hydrogen are plotted in terms of wavelength in Figure 2.3. The Brackett
series evidently overlaps the Paschen and Pfund series. The value of R is the same in Equations
2.1 to 2.5. These observed regularities in the hydrogen spectrum, together with similar
regularities in the spectra of more complex elements, pose a definitive test for any theory of
atomic structure.

2.3 THE BOHR MODEL OF THE ATOM

In 1913, Bohr proposed his theory of atom. He proposed that each electron moves in a fixed
circular orbit around the nucleus. The centripetal force required for the circular motion is
provided by the electrostatic force of attraction between the positively charged nucleus and
negatively charged electron. The energy of electron depends on the radius of the orbit.

In addition, Bohr made two main proposals.



1. The angular moments of the electron are integer multiples of 2𝜋
𝑛ℎ
𝑚𝑣𝑟 = 2𝜋 (𝑛 = 1,2,3 … … … . ) 2.6

Thus, the angular momentum does not have a continuous values i.e. it is quantized. This
means that electrons can have only certain orbital radius, i.e. electrons are allowed to
have only certain values of energy. Electrons can remain only in levels with fixed energy
called energy, levels or stationary states. An electron can remain in an energy level
without losing any energy.

2. An electron can jump from one orbit to another. In doing so it will either lose or gain
energy by emitting or absorbing radiation.
E2 – E1 = hf 2.7
E1 and E2 are energies of energy levels if E1 > E2, then electron will lose energy by
emitting radiation.

Mathematical Treatment of the hydrogen atom according to the Bohr model

24
Consider an electron of mass m and charge e moving with velocity v in a circular orbit of radius
r about a hydrogen nucleus.

1 e2
F= 2.8
4C2 r 2

F = force of attraction on electron due to nucleus.

Centripetal force = mv2/r


Force of attraction = centripetal force
e2 mv 2
=
4e 2 r 2 r

me 2 r
or = (mvr ) 2 (by multiplying with mr3 on both sides)
4 o

From Bohr's first assumption


mvr = nh/2 (n = 1,2,3 …..)

n 2 h 2 0
2
me 2 r  nh 
 =  Or rn = n = 1,2,3…… 2.9
4 o  2  me 2

rn is the orbital radii in the Bohr atom


The radius of the innermost orbit is customarily called the Bohr radius of the hydrogen atom
and is denoted by the symbol a0:

Bohr radius = a0 = r1 = 5.292 x 10-11 m


The other radii are given in terms of a0 by the formula
rn = n2 a0

The total energy of the atom E = Ek + Ep


1 1 e2
Ek = kinetic energy of the electron = mv 2 =  εor
2 2 4
Ep = potential energy of the electron
25
If the nucleus is considered to be a point change, the electric potential at a distance r = e/4εor
Therefore, work done in bringing an electron from  to a point at a distance r from the nucleus
e2
=- εor
4
If the (minus charge is due to because nucleus attracts the electron)
If the potential energy of the electron is taken to be zero at , then
e2
Ep = -  εor
4
1 e2 e2
E = Ek + Ep = −
2 4 o r 4 o r
e2
=- [P.E. = 2K.E.]
8 o r
me 4
E= - 2 2 2 (n = 1, 2, 3 …) 2.10
8 o n h

Remarks: The energy of H atom is always negative.

2.4 ENERGY LEVELS AND SPECTRA


A photon is emitted when an electron jumps from one energy level to a lower level. The various
permitted orbits involve different electron energies. The electron energy En is given in terms of
the orbit radius rn by Equation 1.8 as

𝑒2
𝐸=− 2.11
8𝜋𝜖𝑜 𝑟

Substituting for rn from Equation 2.6, we see that

𝑚𝑒 4 1 𝐸1
Energy levels 𝐸𝑛 = − ( )= 𝑛 = 1,2,3,4 … …. 2.12
8𝜖𝑜 2 ℎ2 𝑛2 𝑛2

𝐸1 = −2.18 × 10−18 𝐽 = −13.6𝑒𝑉

26
The energies specified by Equation 2.12 are called the energy levels of the hydrogen atom and
are plotted in Figure 2.4. These levels are all negative, which signifies that the electron does not
have enough energy to escape from the nucleus. An atomic electron can have only these energies
and no others. An analogy might be a person on a ladder, who can stand only on its steps and not
in between.

Figure 2.4 Energy levels of the hydrogen atom.


27
The lowest energy level E1 is called the ground state of the atom, and the higher levels E2, E3,
E4, . . . are called excited states. As the quantum number n increases, the corresponding energy
En approaches closer to 0. The work needed to remove an electron from an atom in its ground
state is called its ionization energy. The ionization energy is accordingly equal to -E1, the
energy that must be provided to raise an electron from its ground state to an energy of E = 0,
when it is free. In the case of hydrogen, the ionization energy is 13.6 eV since the ground-state
energy of the hydrogen atom is -13.6 eV.

SUMMARY
➢ Spectral line or a line spectra is a dark or bright line. In an otherwise uniform and
continuous spectrum, resulting from emission or absorption of light in a narrow
frequency range of hot solid.
➢ Spectral line (spectroscopy) are often used to identify atoms and molecules from their
characteristic spectral lines.
➢ The spectral series includes, Balmer, Lyman, Paschen, Brackett and Pfund series
➢ Bohr proposed the following postulates:
i. An electron in an atom moves in a circular orbit about the nucleus under the
influence of the Coulomb force between the electron and the nucleus.

ii. An electron moves in an orbit for which its orbital angular momentum L is an
integral multiple of .
iii. An electron moving in an allowed orbit does not radiate electromagnetic energy.
Thus, its total energy E remains constant.
iv. An electron can jump from one orbit to another. In doing so it will either lose or
gain energy by emitting or absorbing radiation.
E2 – E1 = hf
28
𝑚𝑒 4 1 𝐸1
➢ Energy levels 𝐸𝑛 = − ( )= 𝑛 = 1,2,3,4 … …. 2.12
8𝜖𝑜 2 ℎ2 𝑛2 𝑛2

𝐸1 = −2.18 × 10−18 𝐽 = −13.6𝑒𝑉

SELF ASSESSMENT QUESTIONS

(1) Each atom in the periodic table has a unique set of spectral lines. The model of atomic
structure that provides the best explanation for this observation was proposed by
(a) Balmer.
(b) Einstein.
(c) Thomson.
(d) Bohr.

(2) Which one of the following pairs of characteristics of light is best explained by assuming
that light can be described in terms of photons?
(a) photoelectric effect and the effect observed in Young's experiment
(b) diffraction and the formation of atomic spectra
(c) polarization and the photoelectric effect
(d) existence of line spectra and the photoelectric effect

(3) Calculate the wavelength of the lowest energy transition in the Brackett series of the
electronic spectrum of the Li+ ion. The Rydberg constant for He is RLi = 109729 cm-1.
A. 450nm
B. 4050nm
C. 91.1nm
D. 30.3nm
(4) A line in the Paschen series of the emission spectrum of atomic hydrogen is observed at a
wavenumber of 7800 cm-1. Deduce the upper state principal quantum number for this
transition.
A. 7
29
B. 6
C. 4
D. 5
(5) Calculate, from the fundamental constants, the value of the Rydberg constant for a deuterium
atom.
A. 109677cm-1
A. 109707cm-1
B. 109722 cm-1
C. 109729 cm-1
(6) The wavelength of a particular line in the Balmer series is measured to be 379.1 nm. What
transition does it correspond to?

(7) a) The current I due to a charge q moving in a circle with frequency frev is qfrev. Find the
current due to the electron in the first Bohr orbit in A, Amperes.
b) The magnetic moment of a current loop is iA, where A is the area of the loop. Find the
magnetic moment of the electron in the first Bohr orbit in uses of Am2. This magnetic
moment is called the Bohr magneton.

(8) It is possible for a muon to be briefly captured by a proton to form a muonic atom. A muon
is identical to an electron except for its mass, which is 105.7 MeV/c2.
a) Calculate the radius of the first Bohr orbit of a muonic atom.
b) Calculate the magnitude of the lowest energy state.
c) What is the shortest wavelength in the equivalent of the Lyman series for this atom?

REFERENCES
Hugh D. Young and Roger A. Freedman, (2004), University Physics with Modern Physics,11 th
Edition, Pearson Education Publishers

Arthur Beiser (2003), Concepts of Modern Physics, 6th Edition, McGraw-Hill Publishers
30
Ahmad A. Kamal, (2010), 1000 Solved Problems in Modern Physics, Springer Heidelberg
Dordrecht London New York

Should you require more explanation on this study session, please do not hesitate to contact your

e-tutor via the LMS.

Are you in need of General Help as regards your studies? Do not hesitate to contact
the DLI IAG Center by e-mail or phone on:

iag@dli.unilag.edu.ng
08033366677

31
STUDY SESSION 3: NUCLEAR MOTION

INTRODUCTION
Thus far we have been assuming that the hydrogen nucleus remains stationary while the orbital
electron revolves around it. Our major concern in this session is to study the nuclear mass on the
wavelengths of spectral lines.

LEARNING OUTCOMES
At the end of the study session, the students should be able to;
3.1 Explain nuclear motion
3.2 Explain atomic excitation
3.3 Describe the Franck-Hertz experiment

3.1 MEANING OF NUCLEAR MOTION


Consider a system in which both nucleus and electron revolve around their common center of
mass, which is very close to the nucleus because the nuclear mass is much greater than that of
the electron (Figure 3.1).

32
Figure 3.1: The electron and nucleus of a hydrogen atom revolve around a common
center of mass

A system of this kind is equivalent to a single particle of mass m that revolves around the
position of the heavier particle. If m is the electron mass and M the nuclear mass, then m' is given
by equation 3.1.

𝑚𝑀
𝑚′ = 3.1
𝑚+𝑀

The quantity 𝑚′ is called the reduced mass of the electron because its value is less than m.

To take into account the motion of the nucleus in the hydrogen atom, then, all we need do is
replace the electron with a particle of mass 𝑚′ . The energy levels of the atom then become

𝑚′ 𝑒 4 1 𝑚′ 𝐸
Energy levels corrected for nuclear motion = 𝐸𝑛′ = − 2 ℎ2
(𝑛2 ) = ( 𝑚 ) (𝑛12 ) 3.2
8𝜖𝑜

A positronium “atom” is a system that consists of a positron and an electron that orbit each
other. Compare the wavelengths of the spectral lines of positronium with those of ordinary
hydrogen.

Here the two particles have the same mass m, so the reduced mass is


𝑚𝑀 𝑚2 𝑚
𝑚 = = =
𝑚+𝑀 2𝑚 2
33
where m is the electron mass. From Equation 3.2 the energy levels of a positronium “atom” are
𝑚′ 𝐸1 𝐸1
𝐸𝑛′ = ( ) ( 2) = ( 2)
𝑚 𝑛 2𝑛
This means that the Rydberg constant- the constant term in for positronium is half as large as it is
for ordinary hydrogen.

3.2 ATOMIC EXCITATION


There are two main ways in which an atom can be excited to energy above its ground state and
thereby become able to radiate energy. The first process is by collision with another particle in
which part of their joint kinetic energy is absorbed by the atom. Such an excited atom will return
to its ground state in an average of 10-8s by emitting one or more photons (Figure 3.2).

34
Figure 3.2: Excitation by collision

To produce a luminous discharge in a rarefied gas, an electric field is established that accelerates
electrons and atomic ions until their kinetic energies are sufficient to excite atoms they collide
with. Examples of this include Neon signs and mercury-vapor lamps

The second excitation mechanism occurs when an atom absorbs a photon of light whose energy
is just the right amount to raise the atom to a higher energy level.
Example

35
A photon of wavelength 121.7nm is emitted when a hydrogen atom in the n = 2 state drops to
the n = 1 state. Absorbing a photon of wavelength 121.7 nm by a hydrogen atom initially in the n
= 1 state will therefore bring it up to the n = 2 state (Figure 3.3).

Figure 3.3 Origin of emission and absorption spectral lines.

This process explains the origin of absorption spectra. When white light, which contains all
wavelengths, is passed through hydrogen gas, photons of those wavelengths that correspond to
transitions between energy levels are absorbed. The resulting excited hydrogen atoms reradiate
their excitation energy almost at once, but these photons come off in random directions with only
a few in the same direction as the original beam of white light (Figure 3.4).

36
Figure 3.4 The dark lines in an absorption spectrum are never totally dark.

The dark lines in an absorption spectrum are therefore never completely black but only appear
so by contrast with the bright background.

3.3 FRANCK-HERTZ EXPERIMENT

The spectroscopic evidence for the existence of discrete energy levels is supported by the results
of experiments in which electrons are caused to collide with gas atoms. The first successful
experiments of this type were performed by James Franck and Gustav Hertz in 1914. They
performed an experiment on a vacuum tube with a small amount of mercury enclosed using the
apparatus like that shown in figure 3.5.

Electrons which have been emitted thermionically by the filament are accelerated towards the
grid. Then the electron passes through the grid. A series of voltages was applied to the tube. A
small voltage was used to heat a filament for use as an electron source. Three more voltages were
used to establish electric fields inside the tube.

37
Figure 3.5 Apparatus for the Franck-Hertz experiment

The first field is relatively small and its main function is to sweep the electrons away from the
filament. The second field is often regarded as the accelerating field. It gives the electrons the
bulk of their kinetic energy. Also, there is a reverse field that acts to retard electrons from the
counter. If there is only vacuum in the tube then the grid voltage will accelerate electrons to the
counter, and if the retarding voltage is less than the grid voltage a current will be detected.

In the Franck-Hertz experiment, if Va is slightly larger than the retarding voltage, the electrons
have sufficient energy to reach A and a current flows through the galvanometer. If Va is
increased, the current increases at first but as the collision with gas atoms becomes elastic, the
electrons bounce of the atoms without losing energy. As Va is increased further, a point is
reached when the electrons have exactly the right amount of energy to promote the atomic
electrons to higher energy levels and inelastic collisions results.

38
At this point the electrons no longer have sufficient energy to reach the anode and the
galvanometer current falls. If the observed current is plotted against voltage, there will be a
series of peaks and valleys. The peak-to-peak (or valley-to-valley) spacing will correspond to the
excitation energy of the vapor (the multiple valleys are due to multiple inelastic collisions as
shown in Figure 3.5.

Figure 3.5: Results of the Franck-Hertz experiment, showing critical potentials in mercury vapor

SUMMARY
➢ Nuclear motion involves the movement of both nucleus and electron around their
common center of mass, in which the reduced mass concept to validate the motion of the
nucleus in the hydrogen atom.
➢ Reduced mass is a system which is equivalent to a single particle of mass m′ revolving
around the position of the heavier particle.
➢ Atomic excitation is a process where atom in its ground state gains energy to move to
excited states with radiation.
➢ Atom can be excited either by collision with another particle or when an atom absorbs
photon of light whose energy is high enough to raise it from its initial state.

39
➢ Franck-Hertz experiment explains the quantization of energy states of atoms. i.e.
electrons occupied only certain energy levels.

SELF ASSESSMENT QUESTIONS


(1) The quantization of electromagnetic energy is summarized by the equation
a) E = mc
b) E = hω
c) E = hν
d) E = hc

(2) An excited atom is one whose energy state is _____________


a) higher than that of the ground state
b) lower than that of the ground state
c) the same as that of the ground state
d) such that none of the above is correct

(3) The volume of the nucleus of an atom when compared to the extra nuclear part

is_________.

a) bigger
b) smaller
c) same size
d) unpredictable

(4) The fixed circular paths around the nucleus are called_________.

a) orbits
b) orbitals

40
c) nucleons
d) mesons

(5) If an electron of 45 eV had a head-on collision with an Hg atom at rest, what would be
the kinetic energy of the recoiling Hg atom? Assume an elastic collision.
(a) 4.89 × 10−4 𝑒𝑉
(b) 2.00 × 10−6 𝑒𝑉
(c) 4.50 × 10−2 𝑒𝑉
(d) 2.25 × 10−4 𝑒𝑉

(6) Electrons have been removed from a beryllium atom (Z = 4) until only one remains.
Determine the energy of the photon that can be emitted if the remaining electron is in
the n = 2 level.
(a) 13.6 eV
(b) 122 eV
(c) 54.4 eV
(d) 164 eV

(7) Calculate the value of Planck’s constant determined by Franck and Hertz when they
observed the 254-nm ultraviolet radiation using Hg vapor.
(a) 4.13 × 10−4 𝑒𝑉. 𝑠
(b) 4.13 × 10−9 𝑒𝑉. 𝑠
(c) 4.13 × 10−15 𝑒𝑉. 𝑠
(d) 2.54 × 10−9 𝑒𝑉. 𝑠

(8) In the Franck-Hertz experiment, explain why the small potential difference between the
grid and collector plate is useful

REFERENCES

41
Hugh D. Young and Roger A. Freedman, (2004), University Physics with Modern Physics,11 th
Edition, Pearson Education Publishers

Arthur Beiser (2003), Concepts of Modern Physics, 6th Edition, McGraw-Hill Publishers

Stephen T. Thornton and Andrew Rex (2013), Modern Physics for Scientists and Engineers, 4th
Edition, Charles Hartford Publishers

Should you require more explanation on this study session, please do not hesitate to contact your

e-tutor via the LMS.

Are you in need of General Help as regards your studies? Do not hesitate to contact
the DLI IAG Center by e-mail or phone on:

iag@dli.unilag.edu.ng
08033366677

STUDY SESSION 4: LASERS AND MASERS

42
INTRODUCTION
Lasers and Masers are one of the most important inventions of the 19th century. It has found a
great number of important applications in physics, chemistry, biology, medicine, engineering,
telecommunications, computers, warfare and other fields. From the point of view of basic
research, a study of the physical processes which produce the unique properties of laser light are
equally fascinating. The laser is a beautiful example of a system far from thermal equilibrium
which can achieve a macroscopically ordered state through "self-organization". It was the first
example for a non-equilibrium phase transition, and its study eventually gave birth to
synergetics, a new interdisciplinary field of research. The study of lasers and their scientific
applications is often called Quantum Electronics.

LEARNING OUTCOMES
At the end of this study session, students should be able to;
4.1 Explain briefly the meaning and development of lasers
4.2 State the properties of laser
4.3 State the essential parts of a laser
4.4 Principles of a laser
4.5 State five uses of laser

4.1 Meaning and development of Lasers and Masers

The Laser is a light source that produces or amplifies highly directional, coherent radiation of
nearly monochromatic light in the Infra-red, Visible or Ultraviolet regions of the spectrum as a
result of cooperative emission from many atoms. The word "Laser" is an acronym composed of
the initial letters of "Light Amplification by Stimulated Emission of Radiation" while the word
"Maser" is again an acronym standing for "Microwave Amplification by Stimulated Emission of

43
Radiation". Lasers and Masers operate on the same principle that was originally demonstrated in
the microwave domain. When this process of stimulation is applied to optical signals, masers
automatically become lasers. Thus, the Laser principle originates directly from the Maser
principle.

The concept of stimulated emission stems from Einstein when in 1917 he derived Planck's law of
radiation. It took nearly 40 years until it was recognized that this process can be used in a device
producing coherent microwaves and - in particular - a new type of light - laser light. The maser
was proposed by Basov and Prokhorov (1954-1955). Charles H. Townes (1954) and graduate
students James P. Gordon and Herbert J. Zeiger produced the first microwave amplifier, a device
operating on similar principles to the laser. In 1955 Prokhorov and Basov suggested an optical
pumping of multilevel system as a method for obtaining the population inversion, which later
became one of the main methods of laser pumping. Townes, Basov, and Prokhorov shared the
Nobel Prize in Physics in 1964. The extension of this principle to the optical region was done by
Schawlow and Townes (1958). Later in 1960 the Iranian physicist Ali Javan made the first gas
laser using helium and neon. It was the first continuous-light laser. The concept of the
semiconductor laser diode was proposed by Basov and Javan. And the first laser diode was
demonstrated by Robert N. Hall in 1962.

Numerous types of lasers have been invented since then and applied widely. Today they are still
being improved to get better features, such as maximum peak output power and minimum output
pulse duration. As an interesting extension, femtosecond laser can be used to change the color of
metals. This is a pretty new discovery by Chunlei Guo and Anatoliy Vorobyev in 2007.
At this point it will be of interest to you to know what Laser devices looks like.

44
Figure 4.1 Typical Laser device

From the figure above, you will see that it actually looks like our everyday torch which is an
ordinary light source. Although, both are light sources but the principle and properties of a
LASER are totally different from a torch. The properties of LASER beam that makes it different
are elucidated in the next section.
Since the discovery of the laser, literally thousands of types of lasers have been discovered.
However, only a relative few of these lasers have found broadly based, practical applications.
Lasers can be broadly classified into four categories namely:
a) gas discharge lasers
b) semiconductor diode lasers
c) optically pumped lasers,

45
d) and “others,” which involves a category which includes chemical lasers, gas-dynamics
lasers, x-ray lasers, combustion lasers, and others developed primarily for military
applications.

4.2 PROPERTIES OF A LASER LIGHT


The typical properties of laser light make the laser an ideal device for many physical and
technical applications. Let us explain briefly some of its most important properties.

a) Laser light has high directionality: Laser light beam travels straight with almost no
divergence (Figure 4.1). On the other hand, light from an ordinary lamp rays moves in
every direction. A laser with a diameter of a few centimeters can give rise to a laser beam
which, when directed to the moon, gives rise to a spot of a few hundred meters in
diameter. The strict parallelism of the emerging light results in an excellent focusability
which jointly with the high laser light intensity allows a production of very high light
intensities in very small volume elements.

b) Laser light is highly monochromatic: Lasers emit a light beam with a single pure color
with a unique wavelength and frequency (Figure 4.1). On the other hand, the beam of
light emitted from an ordinary light source has many colours mixed together. A single
line in a spectrum of light from a gas discharge is usually considered to be
monochromatic. Widths as narrow as 1Hz has been observed in a laser light. Usually the
spectral width of most Lasers is between 1MHz and 1GHz.

c) Laser light is coherent: Laser beam has a very high coherency resulting in high
amplitudes. When we say a light source is coherent, implies that there is uniform phase of
light relative to time (Figure 4.1). Ordinary light produces an inconsistent phase of light
with respect to time. In other words, an ordinary light is disorganized, not capable of
producing interference. Due to this particular property, light of usual lamps consists of
individual random wave tracks of a few meters length, laser light wave tracks may have a
length of 300,000 km.
46
Some of these specific features are illustrated with diagram in the table below;

Directivity Monochromaticity Coherence


Laser
Beam

Ordinary
Light

Table 4.1: Table depicting the characteristics of a laser beam

d) Brightness: Since the monochromaticity of LASER light is very high, the light energy is
concentrated into a very narrow spectral line width. This implies that the brightness of a
laser beam is very high even if the absolute magnitude of the laser power is low. The
Power (P) of a laser beam is calculated as follows:

𝑃 = 𝑇𝐵 𝐾𝐵 𝛿𝑣 4.1

Where: 𝛿𝑣 is the spectral width and


𝑇𝐵 is the equivalent temperature of a black body to produce the same number of
photons in the same spectral width.

47
If the spectral width of a laser beam is 1MHz with a brightness of temperature of 10 12K What is
the power of the beam?

Using the formular in Equation 4.1, Power (P) = (1 × 106 ) × 1012 = 1𝑀𝑊

e) Laser light can have high intensities. Within laser light pulses, powers far greater than
10'' can be achieved. In order to visualize this power just think that 10' light bulbs, each
with 100 W, are needed to produce the same power.
f) Lasers have Ultra-short Pulses: Lasers can be used to create extremely short pulses by
mode-locking or Q-switching. Laser light can be produced in form of ultrashort pulses of
10-l2s duration (picosecond) or still shorter, e.g. 30 femtoseconds (1 femtosecond= l0-15s).

4.3 ESSENTIAL ELEMENTS OF A LASER

By now you already know the meaning of Lasers. It is imperative for you to know the
constituents of a LASER device. The essential elements of a laser device is represented
schematically in Figure 4.2 consists mainly of three parts. The resonator, an active medium in the
resonator and an energy source for activating the medium. With these components it constitutes a
self-excited oscillator.

48
Figure 4.2: The basic element of a typical laser device

(1) An active medium consisting of a collection of atoms, ions, molecules or even a


semiconductor crystal prepared in a highly non equilibrium state.
(2) A pumping process which supplies energy to excite the constituents of the active medium
into higher quantum mechanical states either pulsed or continuously. The energy must be
supplied at a sufficiently high rate to bring about a special non-thermal equilibrium
condition called population inversion
(3) Suitable optical feedback element which will allow a beam of radiation to pass a several
times through the medium. This means that the beam is actually oscillating within the
medium.

4.4 PRINCIPLE OF A LASER

Laser is based on a principle that involves three main features. These features are;

1) Stimulated Emission
2) Population Inversion
3) An Optical Resonator

4.4.1 Simulated Emission

For you to understand stimulated emission, we shall first consider what spontaneous emission.
There are many energy levels that an electron can occupy but for simplicity we shall consider
only two. If an electron in the excited state with energy E2 spontaneously decays to a ground
state with energy E1 releasing a photon as a difference in energy between the two states. Then
spontaneous emission is said to have taken place. Usually, spontaneous emission produces
fluorescent light. The phase and direction of the photon in spontaneous emission are completely

49
random.

Figure 4.3: Diagram of spontaneous Emission

The energy of the photon is given as follows;

𝐸2 − 𝐸1 = ℎ𝑣

Where; E1 = Energy level 1, also called ground level.

E2 = Energy level 2, also called excited level.

h = Planck constant

v = frequency

On the other hand, if the excited state atom is perturbed by the electric field of a photon with

frequency v, a kind of resonance effect induces the release of a second photon of the same

frequency, direction, polarization and phase with the first photon which is not changed by the

process. This process is called Stimulated emission. For each atom there is one photon before a

stimulated emission and two photons after—thus the name light amplification. Because the two

photons have the same phase, they emerge together as coherent radiation. The laser makes use of

stimulated emission to produce a beam consisting of a large number of such coherent photons.

This is the critical property that allows optical amplification to take place.

50
Figure 4.4: Diagram of stimulated Emission

4.4.2 Population inversion

Although stimulated emission plays a pivotal role in a laser, other factors are also important. For
instance, an external source of energy must be available to excite electrons into higher energy
levels. The energy can be provided in a number of ways, including intense flashes of ordinary
light and high-voltage discharges. If sufficient energy is delivered to the atoms, more electrons
will be excited to a higher energy level than remain in a lower level, a condition known as a
population inversion.

Figure 4.5: (a) Normal population state


(b) Population inversion state

Figure 4.5 compares a normal energy level population with a population inversion. The
population inversions used in lasers involve a higher energy state that is metastable, in the sense
that electrons remain in it for a much longer period of time than they do in an ordinary excited

51
state. The requirement of a metastable higher energy state is essential, so that there is more time
to enhance the population inversion.

4.4.3 Optical resonator

Assuming we are using helium/neon laser similar to Figure 4.2. To sustain the necessary
population inversion, a high voltage is discharged across a low-pressure mixture of 15% helium
and 85% neon contained in a glass tube. The laser process begins when an atom, via spontaneous
emission, emits a photon parallel to the axis of the tube. This photon, via stimulated emission,
causes another atom to emit two photons parallel to the tube axis. These two photons, in turn,
stimulate two more atoms, yielding four photons. Four yield eight, and so on, in a kind of
avalanche. A laser beam is also exceptionally narrow. Since all the power in a laser beam can be
confined to a narrow region, the intensity, or power per unit area, can be quite large. By
coincidence, helium and neon have nearly identical metastable higher energy states, respectively.

4.5 USES OF LASERS


The various fields where laser as found applications include:

a) Communications- Lasers can transmit any type of modulated signal. These are
composed of both audio and video or better known as voice messages and television
signals. Because the laser beam operates at a much higher frequency due to the pulsations
of the beam, they carry far more information than normal sources such as wires, or radio
waves. One beam from the laser can transmit thousands of phone calls and television
programs at the same time. Lasers have very directional beams and therefore can be used
as long distance transmitters. Laser beams only spread slightly as compared to radio
waves. This is like a flashlight compared to a light bulb. For this reason laser beams are
sent through fiber optic cables over great distances with little loss of energy.
b) Construction- Most often the laser is used in construction to measure distances, level
grades, establish heights, level pipes in roads, and even to level land that farmers use by
robotics.

52
c) Manufacturing- The laser is used to cut, weld, heat, and measure.
d) Medicine- Lasers are a useful surgical tool. Surgeons use the heating action of the laser
to burn and remove diseased tissue. This takes only a fraction of the time older methods
did, and does not damage the tissue to the sides. Eye doctors use the laser for various type
of surgery. Not only for cutting out bad parts but to weld the retina when either loose or
broken.
e) Military- Just as radar can be used to determine location and speed so can the laser
beam. The beam is actually bounced off the target and then sent back to where it
originated. Lasers are also used on ships to detect range and distance plus used to help
balance and stabilize.
f) Scientific Research- Lasers are used for many things, from simply shedding light,
heating, cutting and creating super heat surfaces and gases.
g) Multimedia- Lasers are used as bar-code readers, compact disc (CD) and digital video
disc (DVD) players.

SUMMARY

The Laser is a light source that produces or amplify highly directional, coherent radiation of
nearly monochromatic light in the Infra-red, Visible or Ultraviolet regions of the spectrum as a
result of cooperative emission from many atoms.
Lasers can be broadly classified into four categories namely:
a) gas discharge lasers
b) semiconductor diode lasers
c) optically pumped lasers,
d) and “others,” which involves a category which includes chemical lasers, gas-dynamics
lasers, x-ray lasers, combustion lasers, and others developed primarily for military
applications.

53
Laser is based on a principle that involves three main features. These features are;

1) Stimulated Emission
2) Population Inversion
3) An Optical Resonator

SELF ASSESSMENT QUESTION

(1) The frequency of a photon that has an energy of 3.7 10−18 J is __________ s −1 .

a) 5.6 1015
b) 1.8  10 −16
c) 2.5 10−15
d) 5.4 10 −8

(2) The energy of a photon that has a wavelength of 12.3 nm is __________ J.

a) 1.5110−17
b) 4.42 10−23
c) 1.99 10−25
d) 1.62 10−17

(3) Helium-neon laser light (λ = 6.33 x 10-7 m) is sent through a 0.30 mm-wide single slit.
What is the width of the central maximum on a screen 1.0 m from the slit?
a) 2.0 cm
b) 4.2 mm
c) 1.1 cm
d) 2.0 mm
(4) What is the frequency of a helium-neon laser light with a wavelength of 632.8 nm? The speed
of light is 3.00 × 108 m/s.
a) 4.74 × 1014 s-1

54
b) 4.74 × 105 s-1
c) 2.11 × 10-15 s-1
d) 1.58 × 10-15 s-1

(5) Distinguish clearly between spontaneous emission and stimulated emission

REFERENCES

Raymond A. Serway and Chris Vuille, (2012). College Physics, 9th Edition. Charles Hartford
Publishers.

Douglas C. Giancoli, (2009), 4th Edition. Physics for Scientists and Engineers with Modern
Physics, Pearson Educational Publishers.

Hugh D. Young and Roger A. Freedman, (2004), University Physics with Modern Physics,11 th
Edition, Pearson Education Publishers

Should you require more explanation on this study session, please do not hesitate to contact your

e-tutor via the LMS.

Are you in need of General Help as regards your studies? Do not hesitate to contact
the DLI IAG Center by e-mail or phone on:

iag@dli.unilag.edu.ng
08033366677

55
STUDY SESSION 5: QUANTUM PHYSICS

INTRODUCTION
What we have been learning about particle before this stage was classical physics which involves
the treatment of particles as waves. However, theories such as the Bohr Theory cannot explain
why certain spectral lines are more intense than others even though it was able to account for
many aspects of atomic phenomena. It cannot account for the observation that many spectral
lines actually consist of several separate lines whose wavelengths differ slightly. To this end a
more general approach to atomic phenomena is required. This approach was developed in 1925
by Erwin Schrödinger, Werner Heisenberg, Max Born, Paul Dirac, and others under the apt name
of quantum mechanics. “The discovery of quantum mechanics was nearly a total surprise. It
described the physical world in a way that was fundamentally new. This approach, called
quantum mechanics, is the key to understanding the behavior of matter on the molecular, atomic,
and nuclear scales.

LEARNING OUTCOMES
At the end of the study session, students should be able to:
5.1 Explain the concept of quantum physics
5.2 State the wave function
5.3 Give the expression for normalization condition
5.4 State the expression for Schrödinger equation

5.1 WHAT IS QUANTUM MECHANICS


Classical mechanics is an approximation of quantum mechanics. The fundamental difference
between classical (or Newtonian) mechanics and quantum mechanics lies in what they describe.
In classical mechanics, the future history of a particle is completely determined by its initial
position and momentum together with the forces that act upon it. In the everyday world these

56
quantities can all be determined well enough for the predictions of Newtonian mechanics to
agree with what we find. Quantum mechanics arrives at relationships between observable
quantities, but the uncertainty principle suggests that the nature of an observable quantity is
different in the atomic realm. Cause and effect are still related in quantum mechanics, but what
they concern needs careful interpretation. In quantum mechanics the kind of certainty about the
future characteristic of classical mechanics is impossible because the initial state of a particle
cannot be established with sufficient accuracy. For example, instead of asserting, that the radius
of the electron’s orbit in a ground state hydrogen atom is always exactly 5.3 x 10-11m, as the
Bohr theory does, quantum mechanics states that this is the most probable radius.

5.2 WAVE FUNCTION


The quantity with which quantum mechanics is concerned is the wave function ψ of a body.
While ψ itself has no physical interpretation, the square of its absolute magnitude ψ2 evaluated at
a particular place at a particular time is proportional to the probability of finding the body there
at that time. The linear momentum, angular momentum, and energy of the body are other
quantities that can be established from ψ. The problem of quantum mechanics is to determine ψ
for a body when its freedom of motion is limited by the action of external forces. Wave functions
are usually complex with both real and imaginary parts. A probability, however, must be a
positive real quantity. The probability density |ψ|2 for a complex ψ is therefore taken as the
product ψ * ψ of ψ and its complex conjugate ψ*.
The complex conjugate of any function is obtained by replacing i (=√−1) by -i wherever it
appears in the function.

Every complex function ψ can be written in the form


Wave function ψ = A + iB
Where A and B are real functions.
The complex conjugate ψ* of ψ is
Wave function ψ = A - iB
Hence, |ψ|2 = ψ * ψ = A2 - i2B2
57
Since i2 = -1
|ψ|2 = A2 + B2
Therefore |ψ|2 = ψ * ψ is always a real positive quantity.

5.3 NORMALIZATION
Since ψ2 is proportional to the probability density P of finding the body described by ψ, the
integral of |ψ|2 over all space must be finite.


∫−∞|Ψ|2 𝑑𝑉 = 0 5.1

According to equation 5.1, if the particle does not exist, and the integral obviously cannot be ∞
and still mean anything. In addition, |ψ|2 cannot be negative or complex because of the way it is
defined. The only possibility left is that the integral be a finite quantity if ψ is to describe
properly a real body.

It is usually convenient to have |ψ|2 be equal to the probability density P of finding the particle
described by ψ, rather than merely be proportional to P. If |ψ|2 is to equal P, then it must be true
that.
Normalization

∫−∞|Ψ|2 𝑑𝑉 = 1 5.2
Since if the particle exists somewhere at all times,

∫−∞ 𝑃𝑑𝑉 = 1 5.3
Any wave equation that obeys equation 5.2 is said to be normalized.

5.4 SCHRODINGER EQUATION


Schrödinger’s equation, which is the fundamental equation of quantum mechanics in the same
sense that the second law of motion is the fundamental equation of Newtonian mechanics, is a
wave equation in the variable ψ. It is important for us to know that Schrödinger equation can

58
have variety of solutions. Now, we shall discuss the time dependent form of Schrödinger’s
equation.
We have already established the fact that in quantum mechanics the wave function ψ
corresponds to the wave variable y of wave motion in general. However, ψ, unlike y, is not itself
a measurable quantity and may therefore be complex. For this reason we assume that ψ for a
particle moving freely in the +x direction is specified by
𝑥⁄ )
Ψ = 𝐴𝑒 −𝑖𝑤(𝑡− 𝑣 5.4
Take 𝜔 = 2𝜋𝑓 𝑎𝑛𝑑 𝑣 = 𝑓𝜆
−2𝜋(𝑓𝑡−𝑥⁄𝜆)
Ψ = 𝐴𝑒 5.5
This is convenient since we already know what 𝜈 and 𝜆 are in terms of the total energy E and
momentum p of the particle being described by Ψ.
Since 𝐸 = ℎ𝑓 = 2𝜋ℏ𝑓
ℎ 2𝜋ℏ
𝜆= = 5.6
𝑝 𝑝

We obtain
Ψ = 𝐴𝑒 −(𝑖/ℏ)(𝐸𝑡−𝑝𝑥) 5.7
Equation (5.7) describes the wave equivalent of an unrestricted particle of total energy E and
momentum p moving in the +x direction. The expression for the wave function Ψ given by
Equation (5.7) is correct only for freely moving particles. What is of concern to us is an electron
bound to an atom by the electric field of its nucleus. What we must now do is to obtain the
fundamental differential equation for Ψ, which we can then solve for Ψ in a specific situation.
This equation, which is Schrödinger’s equation, can be arrived at in various ways, but it cannot
be rigorously derived from existing physical principles.
To start with, if we differentiate equation 5.7 twice with respect to x, we obtain

𝜕2Ψ 𝑃2
= − ℏ2 Ψ
𝜕𝑥 2
𝜕2 Ψ
𝑃2 Ψ = −ℏ2 𝜕𝑥 2 5.8

Now, if we differentiate equation 5.7 with respect to t, we obtain

59
𝜕Ψ 𝑖𝐸
= − Ψ
𝜕𝑡 ℏ
ℏ 𝜕Ψ
𝐸Ψ = − 𝑖 5.9
𝜕𝑡

At speeds small compared with that of light (𝑣 ≪ 𝑐), the total energy E of a particle is the sum
𝑝2⁄
of its kinetic energy 2𝑚 and its potential energy U, where U is in general a function of
position x and time t:

𝑃2
𝐸= + 𝑈(𝑥, 𝑡) 5.10
2𝑚

For example, in the case of the electron in a hydrogen atom, only the electric field of the nucleus
must be taken into account. Multiplying equation 5.10 through by Ψ gives
𝑃2Ψ
𝐸Ψ = + 𝑈Ψ
2𝑚

To obtain the time dependent form of Schrödinger’s equation, we substitute for EΨ and 𝑃2 Ψ
𝜕Ψ ℏ2 𝜕 2 Ψ
𝑖ℏ = − 2𝑚 𝜕𝑥 2 + 𝑈Ψ
𝜕𝑡

In three dimensions, the time dependent form of Schrödinger’s equation is


𝜕Ψ ℏ2 𝜕2Ψ 𝜕2Ψ 𝜕2Ψ
𝑖ℏ = − 2𝑚 ( 𝜕𝑥 2 + 𝜕𝑦 2 + ) + 𝑈Ψ
𝜕𝑡 𝜕𝑧 2

where the particle’s potential energy U is some function of x, y, z, and t.

Once U is known, Schrödinger’s equation may be solved for the wave function Ψ of a particle,
from which its probability density |Ψ|2 may be determined for a specified x, y, z, t.

SUMMARY

60
➢ Quantum physics is the theoretical basis of modern physics that explains the nature and
behavior of matter and energy on the atomic and subatomic level.
➢ Wave function Ψ of a body is the quantity measuring function in quantum physics, from
which other important measurement like linear momentum, angular momentum, energy
etc can be obtained
➢ Quantum physics field was developed by Max Planck, Albert Einstein, Niels Bohr,
Werner Heisenberg and Erwin Schrodinger.
➢ Schrodinger equation is the fundamental equation in quantum physics for describing
quantum mechanical behavior.

SELF ASSESSMENT QUESTIONS


1) In the probabistic interpretation of wave function Ψ, the quantity |Ψ|2 is___________
a) a probability density.
b) a probability amplitude.
c) a positive probability.
d) a negative probability.

2) The normalization condition is given by


a) ψ * ψ =1
b) ∫ 𝜓 ∗ 𝜓 𝑑𝑥 =1
c) ∫ √𝜓 ∗ 𝜓 = 1

d) √∫ 𝜓 ∗ 𝜓 dx =0

3) Normalize the wave function 𝑒 𝑖(𝑘𝑥−𝑤𝑡) in the region x = 0 to a.

𝑟
4) Normalize the wave function 𝐴𝑟𝑒 −∝ from r = 0 to ∞ where 𝛼 and A are constants.

5) A wave function has the value A sin x between x = 0 and 𝜋 but zero elsewhere. Normalize
the wave function and find the probability that the particle is
61
a) between x = 0 and x = π/4 and
b) between x = 0 and π/2.

6) a) Write down the time-dependent Schrödinger equation for both one dimension and
three dimension and find out its solution by separation of variable.
h) Give the physical interpretation of wave function.

7) If the potential V(x) for a one-dimensional system is independent of time, show that the
expectation value for x is independent of time

REFERENCES
Hugh D. Young and Roger A. Freedman, (2004), University Physics with Modern Physics,11th
Edition, Pearson Education Publishers

Arthur Beiser (2003), Concepts of Modern Physics, 6th Edition, McGraw-Hill Publishers

Stephen T. Thornton and Andrew Rex (2013), Modern Physics for Scientists and Engineers, 4th
Edition, Charles Hartford Publishers

Should you require more explanation on this study session, please do not hesitate to contact your

e-tutor via the LMS.

Are you in need of General Help as regards your studies? Do not hesitate to contact
the DLI IAG Center by e-mail or phone on:

iag@dli.unilag.edu.ng
08033366677

62
STUDY SESSION 6: PARTICLE IN A BOX

INTRODUCTION
The solution of Schrodinger equation requires more elaborate mathematical techniques.
Consequently, a quantum mechanical approach will be used and this requires proficiency in
mathematics. However, since quantum mechanics is the theoretical structure whose results are
closest to experimental reality, we must explore its methods and applications to understand
modern physics. Hence, in this study session, you will study particle in a box and the tunnel
effect.

LEARNING OUTCOMES
At the end of the lesson, the student should be able to explain:
6.1 Particle in a box
6.2 Finite square well

6.1 PARTICLE IN A BOX


The simplest quantum-mechanical problem is that of a particle trapped in a box with infinitely
hard walls. We may specify the particle’s motion by saying that it is restricted to travel along the
x axis between x = 0 and x = L by infinitely hard walls as shown in figure 6.1.

63
Figure 6.1: A square potential well with infinitely high barriers at each end
corresponds to a box with infinitely hard walls.

A particle does not lose energy when it collides with such walls, so that its total energy stays
constant. From a formal point of view the potential energy U of the particle is infinite on both
sides of the box, while U is assumed to be a constant on the inside. Because the particle cannot
have an infinite amount of energy, it cannot exist outside the box, and so its wave function Ψ is 0
for x ≤ 0 and x ≥ L. Our task is to find what Ψ is within the box, namely, between x = 0 and x =
L. The time independent Schrödinger equation for a particle equation moving in one dimension:

ℏ2 𝑑2 𝜓(𝑥)
− 2𝑚 + 𝑉(𝑥)𝜓(𝑥) = 𝐸𝜓(𝑥) 6.1
𝑑𝑥 2

Where:

ℏ (reduced Plank’s constant)
2𝜋
ℎ Plank’s constant (describes size of quanta in quantum mechanics)
𝑚 mass of particle
ψ wave function (replaces the concept of trajectory in classical mechanics)
𝑉(𝑥) potential energy of particle
𝐸 total energy of particle

For a particle in a one-dimensional box of length, 𝐿, the potential energy function is


0 0<𝑥<𝐿
𝑉(𝑥) = { .
∞ elsewhere
This implies that the particle can only exist inside the box where 𝑉(𝑥) = 0.

Therefore:

ℏ2 𝑑 2 ψ 𝑑2 ψ 2𝑚𝐸
− 2𝑚 𝑑𝑥 2 = 𝐸ψ → + ψ=0 6.2
𝑑𝑥 2 ℏ2

.
Let:
64
2𝑚𝐸 𝑑2 ψ
𝑘2 = → + 𝑘2ψ = 0 6.3
ℏ2 𝑑𝑥 2

We have now reduced that equation to a homogeneous second order differential equation with
constant coefficients. We have shown earlier that the general solution to this equation is:
ψ(x) = c1 sin(kx) + c2 cos (kx)
The infinite potential outside of the box implies the following boundary conditions:
ψ(0)=0 and ψ(L)=0
Furthermore, we can consider that ψ=0 for any point outside the box. Applying the first
boundary condition:
ψ(0) = c1 sin(0) + c2 cos(0) = 0 → c2 = 0 → ψ(x) = c1 sin(kx)
Applying the second boundary condition:
𝑛𝜋
c1 sin(kL) = 0 → 𝑘𝐿 = 𝑛𝜋 → 𝑘 =
𝐿
where n is an integer. Therefore:
nπx
ψn (x) = c1 sin ( ) 6.4
L

Note that the subscript on ψ indicates that there are different solutions for different values of n.
We now need to determine c1 . Recall that:
L L L
nπx nπx
|ψn (x)|2 = ∫ ψn (x)2 dx = ∫ c12 sin2 ( 2
) dx = c1 ∫ sin2 ( ) dx
0 0 L 0 L
Since |ψn (x)|2 represents the probability distribution function and we know that the particle will
be somewhere in the box, we know that |ψn (x)|2=1 for 0 < 𝑥 < 𝐿, i.e. there is a 100%
probability that the particle is somewhere inside the box. Therefore:
L
2
nπx
c1 ∫ sin2 ( ) dx =1
0 L
We can show that:
L
nπx 1 sin(nπ) cos (nπ)
∫ sin2 ( ) dx = L ( − )
0 L 2 2nπ
This is the solution as it appears on the TI Voyage 200, but since n is an integer, sin(nπ) = 0.
Hence:

65
L
nπx L
∫ sin2 ( ) dx =
0 L 2

and:

c12 L 2
= 1 → c1 = √
2 L

thus:

2 nπx
ψn (x) = √L sin ( ) 6.5
L

Where n = 1,2,3 …
This is the solution to the wave function for the particle in a one dimensional box.
The normalized wave functions, ψ1 , ψ2 and ψ3 together with the probability densities |ψ1 |2 ,
|ψ2 |2 and |ψ3 |2 are plotted in Figure 6.2.

Figure 6.2: Wave functions and probability densities of a particle confined to a box
with rigid walls.

66
Although ψn may be negative as well as positive, |ψn |2 is never negative and, since ψn is
normalized, its value at a given x is equal to the probability density of finding the particle there.
In every case |ψn |2 = 0 at x = 0 and x = L, the boundaries of the box.

The wave functions shown in Figure 6.2 resemble the possible vibrations of a string fixed at both
ends, such as those of the stretched string of Figure 6.2. This follows from the fact that waves in
a stretched string and the wave representing a moving particle are described by equations of the
same form, so that when identical restrictions are placed upon each kind of wave, the formal
results are identical. We now turn our attention to the total energy.
Recall:
2𝑚𝐸
𝑘2 = 6.6
ℏ2

Since:
𝑛𝜋 ℎ
𝑘= and ℏ = 2𝜋
𝐿

We get:
ℏ2 𝑘 2 ℎ 2 𝑛2 𝜋 2 1 𝑛2 ℎ 2
𝐸= = 4𝜋2 → 𝐸 = 8𝑚𝐿2 6.7
2𝑚 𝐿2 2𝑚

Thus the energy is quantized (since n=1,2,3, … and all other terms are constant). The wave
functions and probability functions are plotted below for a box with length 𝐿 = 1 for
corresponding energy levels.

67
With regards to the wave functions, we define a node as a location other than the endpoint where
ψn (x) = 0. Note that there are 𝑛 − 1 nodes that correspond to each energy level. Consider 𝑛 =
2, there is one node. If we consider the probability distribution function for 𝑛 = 2, we see that it
equals 0 at 1/2. If this is the case, how can a particle get from the left-half to the right-half of
the box? You will discuss a phenomenon called “tunneling” in subsequent physics classes to
explain this behavior.

What is the probability that a particle in the ground state will be found between L/2 and 2L/3?
(note: ground state means 𝑛 = 1)

2
2L/3 2L/3
2 πx 2 2L/3 πx 2
∫ |ψn (x)|2 dx = ∫ √
| sin ( )| dx = ∫ (sin ( )) dx = . 3044 or 30.44%
L/2 L/2 L L L L/2 L

6.2 THE FINITE SQUARE WELL

68
The “box” potential is an oversimplification that is never realized in practice. Given sufficient
energy, a particle can escape the confines of any well. The potential energy for a more realistic
situation, the finite square well, is shown in Figure 6.3,

Figure 6.3: A square potential well with finite barriers. The energy E of the trapped
particle is less than the height U of the barriers.

A classical particle with energy E greater than the well height U can penetrate the gaps at x = 0
and x = L to enter the outer region. Here it moves freely, but with reduced speed corresponding
to a diminished kinetic energy E - U.
A classical particle with energy E less than U is permanently bound to the region 0 < x < L.
Quantum mechanics asserts, however, that there is some probability that the particle can be
found outside this region. That is, the wave function generally is non-zero outside the well, and
so the probability of finding the particle here also is nonzero. For stationary states, the wave
function 𝜓(x) is found from the time independent Schrödinger equation. Outside the well where
U(x) = U, this is equal to
𝑑2 ψ
= 𝛼 2 𝜓(𝑥) 𝑥 < 0 𝑎𝑛𝑑 𝑥 > 𝐿 6.8
𝑑𝑥 2
2𝑚(𝑈−𝐸)
Recall that 𝛼 2 = is a constant
ℏ2

Since U > E, 𝛼 2 necessarily is positive and the independent solutions to this equation are the real
exponentials e+αx and e-αx. The positive exponential must be rejected in region III where x > L to

69
keep 𝜓(x) finite as𝛼 → ∞. In the same way, the negative exponential must be rejected in region I
where x < 0 to keep 𝜓(x) finite as 𝛼 → −∞.. Thus, the exterior wave takes the form 𝜓(𝑥) =
𝐴𝑒 +𝑎𝑥 𝑓𝑜𝑟 𝑥 < 0
𝜓(𝑥) = 𝐵𝑒 −𝑎𝑥 𝑓𝑜𝑟 𝑥 > 𝐿 6.9
The coefficients A and B are determined by matching this wave smoothly onto the wave function
in the well interior. Specifically, we require 𝜓(𝑥) and its first derivative 𝑑𝜓/𝑑𝑥 to be continuous
at x = 0 and again at x = L. This can be done only for certain values of E, corresponding to the
allowed energies for the bound particle. For these energies, the matching conditions specify the
entire wave function except for a multiplicative constant, which then is determined by
normalization.

Figure 6.4: (a) Wave functions for the lowest three energy states for a particle in a
potential well of finite height.
70
(b) Probability densities for the lowest three energy states for a particle in a
potential well of finite height.

Figure 6.4 shows the wave functions and probability densities that result for the three lowest
allowed particle energies. Note that in each case the waveforms join smoothly at the boundaries
of the potential well. The fact that 𝜓 is nonzero at the walls increases the de Broglie wavelength
in the well (compared with that in the infinite well), and this in turn lowers the energy and
momentum of the particle. This observation can be used to approximate the allowed energies for
the bound particle. The wave function penetrates the exterior region on a scale of length set by
the penetration depth, 𝜹, given by

1 ℎ
𝛿= = 6.10
𝛼 √2𝑚(𝑈−𝐸)

At a distance 𝛿 beyond the well edge, the wave amplitude has fallen to 1/e of its value at the
edge and approaches zero exponentially in the exterior region. If it were truly zero beyond this
distance, the allowed energies would be those for an infinite well of length 𝐿 + 2𝛿

𝑛2 𝜋2 ℏ2
𝐸𝑛 ≈ 6.11
2𝑚(𝐿+2𝛿)2

Where n = 1,2, 3 ………

The allowed energies for a particle bound to the finite well are given approximately by Equation
6.11 so long as 𝛿 is small compared with L. The approximation is best for the lowest-lying states
and breaks down completely as E approaches U, where 𝛿 becomes infinite. From this we infer
(correctly) that the number of bound states is limited by the height U of our potential well.
Particles with energies E exceeding U are not bound to the well, that is, they may be found with
comparable probability in the exterior regions.

71
Estimate the ground-state energy for an electron confined to a potential well of width 0.200 nm
and height 100eV.

We estimate the decay length 𝛿 by first neglecting E to get


𝛿=
√2𝑚(𝑈 − 𝐸)
Since E<<U which is 100eV
ℎ 197.3
𝛿= =
√2𝑚(𝑈) √2(511 × 103 )(100)

𝛿 = 0.0195𝑛𝑚

Thus, the effective width of the (infinite) well is 𝐿 + 2𝛿 = 0.239 nm, for which we calculate the
ground-state energy
𝜋 2 197.3
𝐸= = 6.58𝑒𝑉
√2(511 × 103 )(0.239)

From this E we calculate U - E = 93.42 eV and a new decay length


197.3
𝛿= = 0.0202𝑛𝑚
√2(511 × 103 )(93.42100)

SUMMARY
• The solution to the wave function for the particle in a one dimensional box is given as.

72
2 nπx
ψn (x) = √ sin ( )
L L

• The time independent Schrödinger equation for a particle equation moving in one
dimension:
ℏ2 𝑑 2 𝜓(𝑥)
− + 𝑉 (𝑥 )𝜓(𝑥 ) = 𝐸𝜓(𝑥)
2𝑚 𝑑𝑥 2

• The penetration depth, 𝛿, given by


1 ℎ
𝛿= =
𝛼 √2𝑚(𝑈−𝐸)

• The allowed energies for an infinite well is


𝑛 2 𝜋 2 ℏ2
𝐸𝑛 ≈
2𝑚(𝐿 + 2𝛿)2

Where n = 1,2, 3 ………

SELF ASSESSMENT QUESTIONS


A particle is confined to a one-dimensional box on the x-axis between x = 0 and x = L. The
potential height of the walls of the box is infinite. The normalized wave function of the particle,
which is in the ground state, is given by

2 πx
ψn (x) = √ sin ( )
L L

(1) The probability of finding the particle between x = 0 and x = L/3, is closest to________
a) 0.22
b) 0.26
73
c) 0.28
d) 0.20
(2) An electron is in an infinite square well that is 8.9-nm wide. The ground state energy of
the electron is closest to____________
a) 0.0066 eV
b) 0.0085 eV
c) 0.0057 eV
d) 0.0047 eV
(3) An electron is in an infinite square well that is 9.6-nm wide. The electron makes the
transition from the n = 14 to the n = 11 state. The wavelength of the emitted photon is
closest to________
a) 3400 nm
b) 4100 nm
c) 2800 nm
d) 4700 nm
(4) An electron is bound in an infinite square-well potential on the x-axis. The width of the
well is L and the well extends from x = 0 nm to x = 7.3 nm. In a given state, the
normalized wave function of the electron is given by:

2 nπx
ψn (x) = √ sin ( )
L L

The energy of the state is closest to:


a) 0.028 eV
b) 0.035 eV
c) 0.014 eV
d) 0.0071 Ev

(5) Find the probability that a particle trapped in a box L wide can be found between 0.45L
and 0.55L for the ground and first excited states.

74
REFERENCES
Hugh D. Young and Roger A. Freedman, (2004), University Physics with Modern Physics,11th
Edition, Pearson Education Publishers

Arthur Beiser (2003), Concepts of Modern Physics, 6th Edition, McGraw-Hill Publishers

Should you require more explanation on this study session, please do not hesitate to contact your

e-tutor via the LMS.

Are you in need of General Help as regards your studies? Do not hesitate to contact
the DLI IAG Center by e-mail or phone on:

iag@dli.unilag.edu.ng
08033366677

75
STUDY SESSION 7: TUNNELING

INTRODUCTION
Tunneling refers to the quantum mechanical phenomenon where a particle tunnels through
a barrier that it classically could not surmount. It has important applications to modern devices
such as the tunnel diode, quantum computing, and the scanning tunneling microscope. In this
study session, we shall study tunneling through a barrier and the practical applications of
tunneling

LEARNING OUTCOMES
At the end of this study session, students should be able to:
7.1 Explain tunneling through a potential barrier
7.2 State specific applications of tunneling

7.1 TUNNELING THROUGH A POTENTIAL BARRIER

76
Figure 7.1: Wave function 𝜓 for a particle incident from the left on a barrier of height
U and width L.

From figure 7.1 shown above, consider a situation where the potential energy function has a
constant value of U in the region of width L and is zero in all other regions. A potential energy
function of this shape is called a square barrier, and U is called the barrier height. A very
interesting and peculiar phenomenon occurs when a moving particle encounters such a barrier of
finite height and width. Assuming for a particle of energy E, U is incident on the barrier from the
left, the particle is reflected by the barrier according to classical theory. If the particle were
located in region II, its kinetic energy would be negative, which is not classically allowed.
Consequently, region II and therefore region III are both classically forbidden to the particle
incident from the left.

However, according to quantum mechanics, however, all regions are accessible to the particle,
regardless of its energy. From the uncertainty principle, the particle could be within the barrier as
long as the time interval during which it is in the barrier is short. If the barrier is relatively
narrow, this short time interval can allow the particle to pass through the barrier. Let’s approach
this situation using a mathematical representation. The Schrödinger equation has valid solutions
in all three regions. The solutions in regions I and III are sinusoidal. In region II, the solution is
exponential. Applying the boundary conditions, the wave functions in the three regions and their
derivatives must join smoothly at the boundaries. This result is in complete disagreement with
classical physics. The movement of the particle to the far side of the barrier is called tunneling
or barrier penetration.

77
The probability of tunneling can be described with a transmission coefficient T and a reflection
coefficient R. The transmission coefficient represents the probability that the particle penetrates
to the other side of the barrier, and the reflection coefficient is the probability that the particle is
reflected by the barrier. Because the incident particle is either reflected or transmitted, we require
that T + R = 1.
An approximate expression for the transmission coefficient that is obtained in the case of T ≪ 1
is
𝑇 ≈ 𝑒 −2𝐶𝐿 7.1

Where
√2𝑚(𝑈 − 𝐸)
𝐶=

Quantum model of barrier penetration and specifically Equation 7.1 show that T can be non-zero.
That the phenomenon of tunneling is observed experimentally provides further confidence in the
principles of quantum physics.

A 30-eV electron is incident on a square barrier of height 40 eV.


(a) What is the probability that the electron tunnels through the barrier if its width is 1.0 nm?
(b) What is the probability that the electron tunnels through the barrier if its width is
0.10 nm?

1.6×10−19 𝐽
(a) 𝑈 − 𝐸 = 40𝑒𝑉 − 30𝑒𝑉 = 10𝑒𝑉 ( ) = 1.6 × 10−18 𝐽
1𝑒𝑉

√2(9.11 ×10−31 )(1.6 ×10−19 )


2𝐶𝐿 = 2 1.0 × 10−9 = 32.4
1.055 ×10−34
78
𝑇 ≈ 𝑒 −2𝐶𝐿 = 𝑒 −32.4 = 8.5 × 10−15

(b) 2𝐶𝐿 = (0.1)(32.4) = 3.24

𝑇 ≈ 𝑒 −2𝐶𝐿 = 𝑒 −3.24 = 0.039

7.2 APPLICATIONS OF TUNNELING


There are so many applications of tunneling which occurs on atomic and nuclear scale. These
applications include;

7.2.1 Scanning Tunneling Microscope


In 1981, a remarkable new kind of microscope, called the Scanning Tunneling Microscope
(STM), was invented by two IBM researchers, Binnig and Rohrer. It produces images of a
sample by scanning it with a focused beam of electrons. With the STM, one can image atoms on
the surface of any electrically conducting sample such as a metal or semiconductor.
Interestingly, the STM inexpensive and easy to make, and so it quickly became a standard
analytical tool used in thousands of laboratories around the world. In 1984, Binnig and Roher
received the Nobel Prize for the invention of the STM. A schematic of STM is shown in figure
12.2

79
Figure 12.2 Schematic of Scanning Electron Microscope
7.2.2 Resonant tunneling transistors

Figure 12.3: A resonant tunneling transistor

Figure 12.3 shows the addition of a gate electrode at the top of the resonant tunneling device
over the quantum dot. This electrode turns the device into a resonant tunneling transistor. The
basic function of a transistor is amplification, converting a small varying voltage into a large
varying voltage. By applying a small voltage to the gate electrode, the quantized energies can be
brought into resonance with the electron energy outside the well and resonant tunneling occurs.

80
The resulting current causes a voltage across an external resistor that is much larger than that of
the gate voltage; hence, the device amplifies the input signal to the gate electrode.

7.2.3 Cold emission


Cold emission of electrons is relevant to semiconductors and superconductor physics. Cold
emission is similar to thermionic emission where electrons are liberated from the surface of a
metal. Its application is in semiconductor and superconductors. The barrier becomes thin enough
for electrons to tunnel, when the electric field is very large.

7.2.4 Tunnel diode

A diode is a device that allows electric current to flow in one direction. It is made up of N-type
and P-type semiconductor materials which when joined together forms a depletion layer. When
these materials are heavily doped the depletion layer can be thin enough for tunneling. Then,
when a small forward bias is applied the current due to tunneling is significant. As the voltage
bias is increased, the two conduction bands no longer line up and the diode acts typically. Tunnel
diodes can be made with a range of voltages for which current decreases as voltage is increased.

SUMMARY
Tunneling refers to the quantum mechanical phenomenon where a particle tunnels through
a barrier that it classically could not surmount.

The applications of tunneling include:


• Scanning Tunneling Microscope
• Resonant Tunneling Transistors
• Tunneling Diode
• Cold emission

81
SELF ASSESSMENT QUESTIONS
(1) Which of the following statements is not true about quantum tunneling?
a) It is used in modern electronics, and, in fact, our modern-day computers would not
work without it.
b) It plays a crucial role in nuclear fusion in the Sun.
c) It allows electrons and other subatomic particles to pass through wall-like energy
barriers even when it seems they do not have enough energy to get through the
barriers.
d) Although it has been observed to occur, it violates all other known laws of nature, and
explaining it therefore represents a major challenge to physicists.

(2) An electron with kinetic energy 2.80 eV encounters a potential barrier of height Uo =
4.70 eV. If the barrier width is 0.4 nm, what is the probability that the electron will tunnel
through the barrier?
a) 1.1 × 10-1
b) 1.4 × 10-2
c) 2.8 × 10-2
d) 5.5 × 10-2
(3) How does the probability of an electron tunnelling through a potential barrier vary with
the thickness of the barrier?
a) It decreases inversely with thickness.
b) It decreases sinusoidally with thickness.
c) It decreases linearly with thickness.
d) It decreases exponentially with thickness.

(4) When a particle of energy E approaches a potential barrier of height V0, where 𝐸 ≫ 𝑉0,
show that the reflection coefficient is about {[V0 sin(kL)]/2E}2.

(5) Let 12.0eV electrons approach a potential barrier of height 4.2 eV.
82
(a) For what barrier thickness is there no reflection?
(b) For what barrier thickness is the reflection a maximum?

(6) A 1.0-eV electron has a 2.0 x 10-4 probability of tunnelling through a 2.5eV potential
barrier. What is the probability of a 1.0eV proton tunnelling through the same barrier?

(7) What is quantum tunneling?

(8) State practical applications of tunneling

REFERENCES
Hugh D. Young and Roger A. Freedman, (2004), University Physics with Modern Physics,11th
Edition, Pearson Education Publishers
Arthur Beiser (2003), Concepts of Modern Physics, 6th Edition, McGraw-Hill Publishers

Stephen T. Thornton and Andrew Rex (2013), Modern Physics for Scientists and Engineers, 4th
Edition, Charles Hartford Publishers

Should you require more explanation on this study session, please do not hesitate to contact your

e-tutor via the LMS.

Are you in need of General Help as regards your studies? Do not hesitate to contact
the DLI IAG Center by e-mail or phone on:

iag@dli.unilag.edu.ng
08033366677

83
STUDY SESSION 8: TOTAL ANGULAR MOMENTUM

INTRODUCTION
If an atom has an orbital angular momentum and a spin angular momentum due to one or more
of its electrons, we expect that, as is true classically, these angular momenta combine to produce
a total angular momentum. In this session, we examine how the orbital and spin angular
momenta combine and see how this results in energy-level splitting.

LEARNING OUTCOME
At the end of this study session, students should be able to:
8.1 State the total angular momentum of a single-electron atom
8.2 State the total angular momentum of many electron atom
8.3 Itemize the assumptions and limitation of L-S coupling scheme

8.1 SINGLE-ELECTRON ATOMS


Basically before reaching this study session, our discussion center’s only on atoms having a
single electron outside an inert core (for example, the alkalis). For an atom with orbital angular
momentum L and spin angular momentum S, the total angular momentum J is given by

𝐽⃗ = 𝐿
⃗⃗ + 𝑆⃗ 8.1
Since L, Lz, S, and Sz are quantized, the total angular momentum and its z component Jz are also
quantized. If j and mj are the appropriate quantum numbers for the single electron we are

84
considering, quantized values of J and Jz are, in analogy with the single electron of the hydrogen
atom,
𝐽 = √𝐽(𝐽 + 1) ℏ 8.2

𝐽 = 𝑚𝑗 ℏ 8.3
Because 𝑚𝑙 is integral and 𝑚𝑠 is half-integral, 𝑚𝑗 will always be half-integral. Just as the value
of 𝑚𝑙 ranges from -l to l, the value of 𝑚𝑗 ranges from -j to j, and therefore j will be half-integral.

The quantization of the magnitudes of 𝐽⃗, 𝐿


⃗⃗ 𝑎𝑛𝑑 𝑆⃗ are all similar.

𝐽 = √𝑗(𝑗 + 1) ℏ

𝐿 = √𝑙(𝑙 + 1) ℏ

𝑆 = √𝑠(𝑠 + 1) ℏ 8.4
The total angular momentum quantum number for the single electron can only have the values.

𝑗 =𝑙 ±𝑠 8.5
⃗⃗ 𝑎𝑛𝑑 𝑆⃗ are
For an l value of 1, the quantum number j is 3/2 or 1/2, depending on whether 𝐿
aligned or anti-aligned. The notation commonly used to describe these states is
𝑛𝐿𝑗 8.6
where n is the principal quantum number, j is the total angular momentum quantum number, and
L is an uppercase letter (S, P, D, and so on) representing the orbital angular momentum quantum
number.
As discussed previously, the single electron of the hydrogen atom can feel an internal magnetic
field Binternal due to the proton. A careful examination of this effect shows that the spins of the
electron and the orbital angular momentum interact, an effect called spin-orbit coupling. As
usual the dipole potential energy Vsl is equal to− → .→ . The spin magnetic moment is
𝜇𝑠 𝐵𝑖𝑛𝑡𝑒𝑟𝑛𝑎𝑙

⃗⃗ · 𝑆⃗ = SL cosα, where α is the


proportional to -S, and B internal is proportional to L, so that Vsl ~ 𝐿
⃗⃗ 𝑎𝑛𝑑 𝑆⃗. The result of this effect is to make the states with j = l + 1/2 slightly
angle between 𝐿
lower in energy than for j = l + 1/2, because α is smaller when j = l + 1/2. The same applies

85
for the atom when placed in an external magnetic field. The same effect leads us to accept j and
mj as better quantum numbers than ml and ms, even for single electron atoms like hydrogen.

In the absence of an external magnetic field, the total angular momentum has a fixed magnitude
and a fixed z component. Remember that only Jz can be known; the uncertainty principle forbids
Jx or Jy from being known at the same time as Jz. Optical spectra are due to transitions between
different energy levels. For single-electron atoms, we now add the selection rules for Δj. The
restriction of Δl = ±1 will require Δj = ±1 or 0. The allowed transitions for a single-electron atom
are;
Δn = anything Δl = ± 1
Δmj = 0, ± 1 Δj = 0, ± 1 8.7

A comparison between hydrogen and a single-electron atom such as sodium is shown in figure
8.1. The single electron in sodium is 3s1, and the energy levels of sodium should be similar to
those of n = 3 and above for hydrogen.

86
Figure 8.1: The energy-level diagram of sodium (a single electron outside an inert
core) is compared to that of hydrogen. Several allowed transitions are shown for sodium.

However, the strong attraction of the electrons with small l values causes those energy levels to
be considerably lower than for higher l. Note that in Figure 8.1 the 5f and 6f energy levels of
sodium closely approach the hydrogen energy levels, but the 3s energy level of sodium is
considerably lower. The transitions between the energy levels of sodium displayed in Figure 8.1
are consistent with the selection rules of Equation 8.7.

Show that an energy difference of 2 x 10-3 eV for the 3p subshell of sodium accounts for the
0.6-nm splitting of a spectral line at 589.3nm.

The wavelength l of a photon is related to the energy of a transition by

ℎ𝑐
𝐸=
𝜆

For a small splitting, we approximate ΔE by using a differential:

87
−ℎ𝑐
𝑑𝐸 = 𝑑𝜆
𝜆2

Let ΔE = dE and Δ𝜆 = 𝑑𝜆 and taking absolute values yields


−ℎ𝑐
|Δ𝐸| = |Δ𝜆| 8.8
𝜆2

Substituting the values of λ, ΔE and hc into equation 6.8

𝜆2
|Δλ| = |Δ𝐸|
−ℎ𝑐

(589.3𝑛𝑚)2 (2 x 10−3 eV )
|Δλ| = = 0.6nm
1.24 × 103 𝑒𝑉. 𝑛𝑚

The value of 0.6 nm agrees with the experimental measurement for sodium.

8.2 MANY-ELECTRON ATOMS


The interaction of the various spins and angular momenta becomes formidable for more than two
electrons outside an inert core. Various empirical rules help in applying the quantization results
to such atoms. The best-known rule set is called Hund’s rules, introduced in 1925 by the German
physicist Friedrich Hund (1896– 1997), who is known mainly for his work on the electronic
structure of atoms and molecules. We consider here the case of two electrons outside a closed
shell (for example, helium and the alkaline earths). The order in which a given subshell is filled
is governed by Hund’s rules:

Rule 1. The total spin angular momentum S should be maximized to the extent possible without
violating the Pauli Exclusion Principle.
Rule 2. Insofar as rule 1 is not violated, L should also be maximized.
Rule 3. For atoms having subshells less than half full, J should be minimized.

88
For example, the first five electrons to occupy a d subshell should all have the same value of ms.
This requires that each one has a different ml (because the allowed ml values are (-2, -1, 0, 1, 2).
By rule 2 the first two electrons to occupy a d subshell should have ml = 2 and ml = 1 or ml = -2
and ml = -1.
At this point, we shall now discuss the spin-spin and orbital-orbital interactions. For the two-
electron atom, we label the electrons 1 and 2 so that we have ⃗⃗⃗⃗⃗
𝐿1 · ⃗⃗⃗⃗
𝑆1 and ⃗⃗⃗⃗⃗
𝐿2 · ⃗⃗⃗⃗
𝑆2 . The total
angular momentum J is the vector sum of the four angular momenta:

𝐽⃗ = ⃗⃗⃗⃗⃗
𝐿1 + ⃗⃗⃗⃗⃗
𝐿2 + ⃗⃗⃗⃗
𝑆1 + ⃗⃗⃗⃗
𝑆2 8.9

The next important concept we shall consider are the two coupling schemes, called LS coupling
and jj coupling, for combining the four angular momenta to form J. The decision of which
scheme to use depends on relative strengths of the various interactions. We shall see that jj
coupling predominates for heavier elements.

8.3 THE L-S COUPLING SCHEME: ITS ASSUMPTION AND LIMITATION


When we derive the electronic states arising from a given atomic configuration, we take
advantage of the simplification that all the complete shells of inner core electrons contribute no
net orbital and spin angular momenta. Apart from the attraction between the nucleus and
electrons, there remain two principal contributions to the energy in an atomic system:
(1) inter-electronic repulsion, which is responsible for the energy separation between the terms
arising from a given configuration, and
(2) spin-orbit interaction, which gives rise to the energy intervals between the components within
a given term. In the L-S coupling scheme, it is assumed that electronic repulsion is much greater
than spin-orbit interaction. In the terminology of quantum mechanics, the spin-orbit interaction
in this scheme is treated as a perturbation.

The L-S coupling scheme involves the orbital angular momentum, quantum number L and the
spin angular momentum quantum number S. It is conventionally designated by a term symbol

89
2S+1
written in the form LJ. The addition of L and S gives the total angular momentum quantum
number J, which has the allowed values given by the sequence: L + S, L + S – 1, … , |L – S|. This
results in a multiplicity of 2S + 1 for a given orbital angular momentum L (except when L = 0)
which is written as a subscript preceding the code symbol S, P, D, F, … for L = 0, 1, 2, 3, … .
Beware of the possible confusion between the code symbol S for L = 0 and the total spin angular
momentum quantum number, which is also designated as S. Spin-orbit interaction between the
total orbital and spin angular momenta give rise to different energies for each allowed value of J.
For the lighter elements, these energy separations are relatively small.

When atomic states are accurately represented by Russell-Saunders coupling, the energy
ordering of different terms arising from a given electron configuration follow Hund's rules stated
previously.

Let us now explore the quantitative validity of the Russell-Saunders coupling scheme. How well
does it hold and when does it begin to break down? These questions can be answered by
referring to some atomic spectral data. For example, the atomic energy levels arising from
configurations np3, n = 2 – 6, of Group 15 elements. Applying the L-S coupling scheme to the p3
configuration, we get three spectroscopic terms , namely 4S, 2D, and 2P, which give rise to five
energy levels, 4S1½ (ground state), 2D1½, 2D2½, 2P½, and 2P1½.

So how good is the assumption of the L-S coupling scheme that electronic repulsion is much,
much larger than the spin-orbit interaction? If we look at the data for the nitrogen atom, we find
the two energy gaps between the three terms (which give a measure of the electronic repulsion)
are of the order of 104 cm-1. On the other hand, the intervals between the two states arising from
the same term (which accounts for the spin orbit interaction within the term) are only a few cm-1.
Based on these results, the aforementioned assumption is clearly justified. For the other four
atoms within the same Group, the two energy gaps among the three terms remain to be about 104
cm-1. The spin-orbit interaction increases by about an order of magnitude as we go down the
Periodic Table. Also, for the lighter elements, above the ground state 4S2½, we can clearly see

90
two sets of “doublet” levels, with very small to relatively small intervals between the two levels.
But such a description no longer exists as we go to the sixth row of the Periodic Table.

Before leaving this section, it should be noted that the breakdown of the L-S coupling scheme for
the heavier elements is by no means merely an empirical trait. Rather, it has a theoretical basis.
The spin-orbit interaction energy in a hydrogen-like atom can be calculated exactly and it is
proportional to the ratio of Z4/n3, where Z is the atomic number and n the principal quantum
number of the outer electrons. It is expected that this proportionality holds approximately for
other atoms in the Periodic Table.

jj COUPLING
This coupling scheme predominates for the heavier elements, where the nuclear charge causes
the spin-orbit interactions to be as strong as the forces between the individual Si and the
individual Li. The coupling order becomes

⃗⃗⃗
𝐽1 = ⃗⃗⃗⃗⃗
𝐿1 + ⃗⃗⃗⃗
𝑆1
⃗⃗⃗⃗
𝐽2 = ⃗⃗⃗⃗⃗
𝐿2 + ⃗⃗⃗⃗
𝑆2 8.10
And then
𝐽⃗ = ∑𝐼 ⃗𝐽⃗𝑖 8.11
The spectroscopic or term notation is also used to describe the final states in this coupling
scheme.

Intermediate coupling
For a few atoms, the correction to the electrostatic terms, V is approximately equal to the
magnetic interactions, P (i.e. 𝑃 ≈ 𝑉) this is known as intermediate coupling. Here, none of the
⃗⃗ , ⃗⃗⃗⃗⃗⃗
operations 𝐿 ⃗⃗⃗⃗⃗⃗𝑠 , ⃗⃗⃗⃗
𝑀𝐿 , 𝑆⃗, 𝑀 𝐽12 commute with the Hamiltonian.

SUMMARY
Total angular momentum J is given by
91
𝐽⃗ = 𝐿
⃗⃗ + 𝑆⃗

The quantization of the magnitudes of 𝐽⃗, 𝐿


⃗⃗ 𝑎𝑛𝑑 𝑆⃗are all similar.

𝐽 = √𝑗(𝑗 + 1) ℏ

𝐿 = √𝑙(𝑙 + 1) ℏ

𝑆 = √𝑠(𝑠 + 1) ℏ

Hund’s rules:
Rule 1. The total spin angular momentum S should be maximized to the extent possible without
violating the Pauli Exclusion Principle.
Rule 2. Insofar as rule 1 is not violated, L should also be maximized.
Rule 3. For atoms having subshells less than half full, J should be minimized

SELF ASSESSMENT QUESTIONS

(1) Which quantum numbers must be the same for the orbitals that they designate to be
degenerate in a many-electron system?
a) ms only
b) n, l, and ml
c) n and l only
d) n, l, ml, and ms

(2) Which one of the following represents an acceptable set of quantum numbers for an electron
in an atom? (arranged as n, l, ml , and ms)
a) 5, 4,- 5, 1/2
b) 3, 3, 3, -1/2
c) 2, 2, -1, -1/2
d) 1, 0, 0, ½

92
(3) If the zirconium atom ground state has S = 1 and L = 3,
(a) What are the permissible values of J?
(b) Write the spectroscopic notation for these possible values of S, L, and J. Which one of
these is likely to represent the ground state?

(4) What are S, L, and J for the following states?


(a) 1S0
(b) 2D5/2
(c) 5F1
(d) 3F4

(4) State the three rules of Hund’s

(5) What are the possible orientations of J for the j = 3/2 and j = 1/2 states that correspond to l =
1?

(6) What is the energy difference between a spin-up state and spin-down state for an electron in
an s state if the magnetic field is 2.55 T?

REFERENCES
Hugh D. Young and Roger A. Freedman, (2004), University Physics with Modern Physics,11 th
Edition, Pearson Education Publishers

Arthur Beiser (2003), Concepts of Modern Physics, 6th Edition, McGraw-Hill Publishers

Stephen T. Thornton and Andrew Rex (2013), Modern Physics for Scientists and Engineers, 4th
Edition, Charles Hartford Publishers

93
Should you require more explanation on this study session, please do not hesitate to contact your

e-tutor via the LMS.

Are you in need of General Help as regards your studies? Do not hesitate to contact
the DLI IAG Center by e-mail or phone on:

iag@dli.unilag.edu.ng
08033366677

STUDY SESSION 9: HYDROGEN ATOM

INTRODUCTION
As the simplest atom and the only atom for which analytic wave functions can be derived,
hydrogen is clearly the foundation of all atomic physics. You have already gone over the solution
of Schrodinger’s equation in the Quantum Physics course and so the presentation here will not
cover all the details of the mathematical solution of the equations, and will instead focus on
interpreting and understanding the results. In this study session, we shall see how Schrödinger’s
quantum theory of the hydrogen atom achieves its results.

LEARNING OUTCOMES
At the end of this study session, student should be able to:
9.1 Solve the schrodinger equation
9.2 State the Bohr Magneton
9.3 Explain the Normal Zeeman effect

94
9.1 SOLUTION OF SCHRODINGER’S EQUATION
The time independent Schrodinger’s equation for the hydrogen atom is

− 2 2 e2 
  −  (r) = E (r)
 2 4 o r 
9.1

where  is the so called reduced mass given by  = me m p /(me + m p )

me is the mass of the electron


m p is the mass of the proton,

r is the vector between the electron and the proton.

There are a number of implicit assumptions in using the time independent Schrodinger’s
equation, in particular, Schrodinger’s equation is non-relativistic and we assume that the proton
is a point-like particle. It turns out that these assumptions lead to small errors in the final result
and we will look at this so called fine structure later on in the course.
In order to fully define the problem we also require boundary conditions. These are based
on the physically reasonable assumptions that the wavefunction must tend to zero as r tends to
infinity and the wavefunction must be single valued.
One important feature of the above Schrodinger equation is that the electrostatic potential
is a so called central potential, i.e. it depends only on the distance from the origin and not on the
direction in space. Another way of saying this is that the potential is spherically symmetric.
These two identical statements strongly suggest that we should solve this problem using polar
coordinates. In polar co-ordinates the Laplace operator  2 is written

1  2  1     1 2
2 =  r  +  sin   +
r 2 r  r  r 2 sin      r 2 sin 2   2 9.2

which can be rewritten using the total orbital angular momentum operator as

95
1  2  1 2
2 =  r  − 2 2 L̂
r r
2
r r

The fact that the radial and angular derivatives are in separate terms in the Laplace operator
suggests that we trial a separation of variables solution of the form

 (r, ,  ) = R(r)Y ( ,  ) 9.3

Substituting this trial solution into the Schrodinger equation and rearranging gives
1    2   2E 2 2  e2  1 1
  r  + r + r  R(r) = 2 L̂2Y ( ,  )
R(r)  r r 2 2
4 o  Y ( ,  )
9.4

As the two sides of this equation do not depend on the same variables then for the equation to be
true they must both equal something which doesn’t depend on any variables, which is a long way
of saying a constant. If we set them both equal to a constant b then we have

1    2   2E 2 2  e2 
 r + 2 r + 2 r  R(r) = b
R(r)  r  r  4 o 
9.5
1 1
Lö2Y ( ,  ) = b
2
Y ( ,  ) 9.6

which can be rearranged to show that they are in fact that definitions of eigenfunctions of two
different operators.

In fact the solutions to the second equation are the spherical harmonics and require two quantum
numbers to define them l, the total orbital angular momentum quantum number, and m, the
magnetic quantum number, i.e.

96
Yl ,m ( ,  ) = Pl m (cos )eim
9.7

and this equation gives us that b equals l(l+1) where l is the total orbital angular momentum
quantum number.

If we substitute b=l(l+1) into the first of the equations above and rearrange we obtain the
equation

 2 1   2  e2 h 2 l (l + 1) 

 2  r 2 r  r  4 r 2  r 2  R ( r ) = ER ( r )
r − +
   o  9.8

Thus, if l is not equal to zero, then the orbital angular momentum term is effectively a short
range repulsive potential. This is the quantum equivalent of the centrifugal potential.

The solutions to these equations are in the form of a polynomial of r multiplied by an


exponential of r which decays as r tends to infinity. These solutions are clearly dependent on the
quantum number l but are also dependent on a second quantum number n which is called the
principle quantum number, and can be any positive integer, i.e. n = 1,2,3, . It turns out that in
order for these solutions to obey the boundary conditions as r tends to infinity that

l  n −1

Some examples of the mathematical form of these eigenfunctions are

97
R1,0 = 2 ( a0 ) exp ( − r / a0 )
−3/ 2

R2,0, = 2 ( 2 a0 ) (1 − r / 2a0 ) exp ( − r / 2a0 )


−3/ 2

R2,1 = 2 ( 3 ) ( 2a0 ) ( r / 2a0 ) exp ( − r / 2a0 )


−3/ 2

R3,0 = 2 ( 3 a0 )
−3/ 2
(1 − 2r / 3a 0 + 2 r 2 / 27a0 2 ) exp ( − r / 3a0 )

(
R3,1 = 4 2 3 ( 3 a0 ) ) −3/ 2
( r / 3a0 ) (1 − r / 6a0 )exp ( − r / 3a0 )

Where the first subscript of the R-function is the principle quantum number n and the second the
total angular momentum number l, and
 2 o
ao =  5.3  10 −11 m
2e 2
It is helpful for dealing with these functions to have some idea of their form which are shown in
the figures below.

0.6

n =1, l= 0
0.4
Pn,l (r/a0 )

n =2, l= 0
0.2 n =3, l= 0

0
0 5 10 15 20
r/a 0

2
Figure 9.1: Radial extent of the probability density Pn ,l ( r ) = Cn ,l r 2 Rn ,l ( r ) for the

hydrogen atom with n = 1, 2, 3 and l = 0.

98
0.6

0.5

n= 1, l= 0
0.4

0.3
Rnl (r )

n= 2, l= 0
0.2
n= 2, l= 1
0.1

0
0 2 4 6 8 10
-0.1
r/a 0

Figure 9.2: Radial part of the wavefunction Rn ,l ( r ) for n = 1, l = 0; n = 2, l = 0, 1.

99
0.15
n =3, l= 1

0.1 n =3, l= 2 n =3, l= 0


Pn,l (r/a0 )

0.05

0
0 5 10 15 20
r/a 0

2
Figure 9.3: Radial extent of the probability density Pn ,l ( r ) = Cn ,l r 2 Rn ,l ( r ) for the

hydrogen atom with n = 3, and l = 0, 1, 2.

It should be noted that

i) orbitals with larger n extend further from the nucleus,


ii) that the probability density for finding an electron at the nucleus tends to zero in all
cases, and that the wavefunction tends to zero for all wavefunctions except the l=0
wavefunctions,
iii) there are n − l − 1 nodes in the radial wavefunction.

The energy associate with these wavefunctions is given by

2
1 2  e 2  1 13.6
En = − 
2 
 2  − 2 eV
4  4 o  n n
9.9

100
Which is often presented in the form

 2 c 2
En = −
2n 2
where  is the so called fine structure constant, and is given by
e2 1
=  9.10
4 o c 137

Thus, the energy of the eigen wavefunctions of the hydrogen atom depend solely on the principal
quantum number n. The fact that the energy does not depend on l is what is called an accidental
degeneracy and depends on the functional form of the Coulomb potential. The fact that the
energy does not depend on m, the magnetic quantum number, is due to the rotational symmetry
of the hydrogen atom.

9.2 BOHR MAGNETON


Microscopic magnetic moments

Motion of charged particles inside atoms gives rise to microscopic magnetic moments. These are
related to the angular momentum of the particle.

For a particle with momentum p = mu , the angular

momentum about a point O is L=r´p

where r is the vector pointing from O to the


position of the particle.

For a particle travelling with constant speed u in a circle of radius r , the magnitude of the
angular momentum about the center of the circle is

101
L = mru

The direction is the same as the direction of advance of a right hand screw turned in the direction
of the velocity. The frequency of the motion is

u
f= 9.11
2p r

If the particle carries the charge q , the circular motion gives rise to a current

qu
I = qf = 9.12
2p r
A magnetic moment is given as

qu r qmru q
m = p r2I = = = L 9.13
2 2m 2m

Taking the direction into account, we find

q
m= L 9.14
2m

If the charged particle is electron, the magnetic moment is

e
mL = - L 9.15
2me

In quantum mechanics, the angular momentum is found to be an integral multiple of Planck


constant , which has the value

102
h
= =1.05´10-34 J × s
2p

Expressing angular momentum is units of , we can write

L
m L = -m B 9.16

The quantity 𝜇𝐵 is called the Bohr magneton and is given as


𝑒ℏ
𝜇𝐵 = 9.17
2𝑚𝑒

e
B = = 9.27  10 −24 A  m 2 = 9.27  10 −24 J / T = 5.79  10 −5 eV / T
2me

The Bohr magneton gives an estimate of the size of microscopic magnetic moment due to
electrons.

9.3 ZEEMAN EFFECT


Zeeman Effect boarders on how atoms interact with a magnetic field. In an external magnetic
field B, a magnetic dipole has an amount of potential energy Um that depends upon both the
magnitude µ of its magnetic moment and the orientation of this moment with respect to the field.
(Figure 9.4).

103
Figure 9.4: A magnetic dipole of moment µ at the angle θ relative to a magnetic field
B.

The torque 𝜏 on a magnetic dipole in a magnetic field of flux density B is


𝜏 = 𝜇𝐵𝑠𝑖𝑛𝜃 9.18
where 𝜃 is the angle between 𝜇 and B.

The torque is a maximum when the dipole is perpendicular to the field, and zero when it is
parallel or antiparallel to it. To calculate the potential energy Um we must first establish a
reference configuration in which Um is zero by definition. It is convenient to set Um = 0 when
𝜃 = 𝜋⁄2 = 900 that is, when 𝜇 is perpendicular to B. The potential energy at any other
orientation of 𝜇 is equal to the external work that must be done to rotate the dipole from 𝜃0 =
𝜋⁄2 to the angle 𝜃 that corresponds to that orientation.
Therefore
𝜃 𝜃
𝑈𝑚 = ∫𝜋⁄2 𝜏𝑑𝜃 = 𝜇𝐵 ∫𝜋⁄2 𝑠𝑖𝑛𝜃𝑑𝜃 = −𝜇𝐵𝑐𝑜𝑠 𝜃 9.19

When µ points in the same direction as B, then Um = -µB, its minimum value.

The magnetic moment of the orbital electron in a hydrogen atom depends on its angular
momentum L. Hence both the magnitude of L and its orientation with respect to the field
determine the extent of the magnetic contribution to the total energy of the atom when it is in a
magnetic field. The magnetic moment of a current loop has the magnitude
𝜇 = 𝐼𝐴 9.20
where
I = current
A = area of enclosure.
An electron that makes f rev/s in a circular orbit of radius r is equivalent to a current of –ef.

104
Therefore, its magnetic moment is
𝜇 = −𝑒𝑓𝜋𝑟 2 9.21
Since the linear speed v of the electron is 2πfr its angular momentum is
𝐿 = 𝑚𝑣𝑟 = 2𝜋𝑚𝑓𝑟 2 9.22
Figure 9.5 shows the comparison between equation 9.21 and 9.22.
Electron magnetic moment is
𝑒
𝜇 = −( )𝐿
2𝑚

Figure 9.5: (a) Magnetic moment of a current loop enclosing area A.


(b) Magnetic moment of an orbiting electron of angular momentum L.

The quantity (e/2m), which involves only the charge and mass of the electron, is called its
gyromagnetic ratio. The minus sign means that µ is in the opposite direction to L and is a
consequence of the negative charge of the electron. While the above expression for the magnetic
moment of an orbital electron has been obtained by a classical calculation, quantum mechanics
yields the same result. The magnetic potential energy of an atom in a magnetic field is
𝑒
𝑈𝑚 = (2𝑚) 𝐿𝐵𝑐𝑜𝑠𝜃 9.23

Which depends on both B and θ


105
In a magnetic field, then, the energy of a particular atomic state depends on the value of ml as
well as on that of n. A state of total quantum number n breaks up into several sub-states when the
atom is in a magnetic field, and their energies are slightly more or slightly less than the energy of
the state in the absence of the field. This phenomenon leads to a “splitting” of individual spectral
lines into separate lines when atoms radiate in a magnetic field. The spacing of the lines depends
on the magnitude of the field. The splitting of spectral lines by a magnetic field is called the
Zeeman Effect after the Dutch physicist Pieter Zeeman, who first observed it in 1896. The
Zeeman effect validates the concept of space quantization.

Since, ml can have the 2l + 1 values of +l through 0 to -l, a state of given orbital quantum
number l is split into 2l + 1 sub-states that differ in energy by 𝜇𝐵 𝐵 when the atom is in a
magnetic field. However, because changes in ml are restricted to Δml = 0, ±1, we expect a
spectral line from a transition between two states of different l to be split into only three
components, as shown in Figure 9.6. The normal Zeeman effect consists of the splitting of a
spectral line of frequency 𝑓𝑜 into three components whose frequencies are
𝐵 𝑒
𝑓1 = 𝑓0 − 𝜇𝐵 ℎ = 𝑓0 − 𝐵
4𝜋𝑚

𝑓2 = 𝑓0
𝐵 𝑒
𝑓3 = 𝑓0 + 𝜇𝐵 ℎ = 𝑓0 + 𝐵 9.24
4𝜋𝑚

106
Figure 9.6: The normal Zeeman effect a spectral line of frequency 𝑓0 is split into three
components

A sample of a certain element is placed in a 0.300-T magnetic field and suitably excited. How far
apart are the Zeeman components of the 450-nm spectral line of this element?

107
The separation of the Zeeman component is given as
𝑒
𝛥𝑓 = 𝐵
4𝜋𝑚
𝑐
Since = ,
𝜆
𝑑𝜆
𝑑𝑓 = 𝑐
𝜆2

𝜆2 𝛥𝑓 𝑒𝐵𝜆2
𝛥𝜆 = =
𝑐 4𝜋𝑚𝑐
−7 2
(1.6 ×10−19 )(0.300)(4.50 ×10 )
=
4𝜋(9.11×10−31 )(3.0×108 )

=2.83 × 10−12 𝑚

SUMMARY
The energy associate with these wavefunctions is given by
2
1 2  e 2  1 13.6
En = − 
2 
 2  − 2 eV
4  4 o  n n

The quantity 𝜇𝐵 is called the Bohr magneton and is given as


𝑒ℏ
𝜇𝐵 =
2𝑚𝑒
The splitting of spectral lines by a magnetic field is called the Zeeman Effect
The Normal Zeeman effect consists of the splitting of a spectral line of frequency 𝑓𝑜 into three
components whose frequencies are
𝐵 𝑒
𝑓1 = 𝑓0 − 𝜇𝐵 ℎ = 𝑓0 − 𝐵
4𝜋𝑚

𝑓2 = 𝑓0

108
𝐵 𝑒
𝑓3 = 𝑓0 + 𝜇𝐵 ℎ = 𝑓0 + 𝐵
4𝜋𝑚

SELF ASSESMENT QUESTION


(1) According to the quantum mechanical picture of the atom, which one of the following
is a true statement concerning the ground state electron in a hydrogen atom?
(a) The ground state electron has zero kinetic energy.
(b) The ground state electron has zero binding energy.
(c) The ground state electron has zero ionization energy.
(d) The ground state electron has zero orbital angular momentum.

(2) A hydrogen atom is in a state for which the principle quantum number is n = 3. How
many possible such states are there for which the magnetic quantum number is m = 0?

(a) 2
(b) 6
(c) 10
(d) 4

(3) According to the quantum mechanical picture of the atom, which one of the following
statements is true concerning the magnitude of the angular momentum L of an
electron in the n = 3 level of the hydrogen atom?
(a) L is 0.318h.
(b) L could be 0.225h or 0.276h.
(c) L could be 0.225h or 0.390h.
(d) L could be 0.159h or 0.318h.

(4) Find the minimum magnetic field needed for the Zeeman effect to be observed in a
spectral line of 400-nm wavelength when a spectrometer whose resolution is 0.010 nm is
used.
(a) 1.00T

109
(b) 1.34T
(c) 4.00T
(d) 0.40T

(5) Derive the expression for the Bohr Magneton and calculate its value.

(6) State the frequencies of the three components obtained from splitting the spectral line
of frequency fo due to normal Zeeman effect.
(7) In the Bohr model of the hydrogen atom, what is the magnitude of the orbital magnetic
moment of an electron in the nth energy level?
(8) Show that the magnetic moment of an electron in a Bohr orbit of radius rn is proportional
to √𝑟𝑛

REFERENCES
Hugh D. Young and Roger A. Freedman, (2004), University Physics with Modern Physics,11th
Edition, Pearson Education Publishers

Arthur Beiser (2003), Concepts of Modern Physics, 6th Edition, McGraw-Hill Publishers

Stephen T. Thornton and Andrew Rex (2013), Modern Physics for Scientists and Engineers, 4th
Edition, Charles Hartford Publishers

Should you require more explanation on this study session, please do not hesitate to contact your

e-tutor via the LMS.

Are you in need of General Help as regards your studies? Do not hesitate to contact
the DLI IAG Center by e-mail or phone on:

110
iag@dli.unilag.edu.ng
08033366677
STUDY SESSION 10: QUANTUM NUMBERS

INTRODUCTION
When the Schrodinger’s wave equation is solved, it yields many orbital, each of which is
characterized by a set of three numbers called quantum numbers, which describes the energy,
size, shape, and the number of orbital of each type. They are called the principal quantum
number n, the angular momentum quantum number l, and the magnetic quantum number ml. The
first part of this study session will be to solve the time dependent Schrodinger equation and then
use it to explain the quantum numbers

LEARNING OUTCOMES
At the end of this study session, the student should be able to:
10.1 Solve the Schrodinger equation for hydrogen atom
10.2 Explain briefly the quantum numbers

10.1 SOLUTION OF THE SCHRODINGER EQUATION FOR HYDROGEN ATOM


The potential energy for the hydrogen atom is given as follows

𝑘𝑒 2
𝑉(𝑟) = − 10.1
𝑟

The time independent Schrodinger equation in spherical coordinates is:

ℏ2 1 𝜕 𝜕𝜓 1 1 𝜕 𝜕𝜓 1 𝜕2𝜓
𝐸𝜓 = − 2𝜇 {𝑟 2 𝜕𝑟 (𝑟 2 𝜕𝑟 ) − 𝑟 2 [𝑠𝑖𝑛𝜃 𝜕𝜃 (𝑠𝑖𝑛𝜃 𝜕𝜃 ) + 𝑠𝑖𝑛2 𝜃 𝜕𝜙2 ]} + 𝑉(𝑟)𝜓 10.2

111
Separating the variables:
𝜓(𝑟, 𝜃, 𝜙) = 𝑅(𝑟)𝑓(𝜃)𝑔(𝜙)

𝑔(𝜙) = 𝑒 𝑖𝑚𝜙

Where 𝐶𝜙 = 𝑚2 , 𝑚 = 0, ±1, ±2 …

Lets consider m = 0. The simplest solution for 𝑓(𝜃) is:

𝑓(𝜃) = 𝐶
Then
C𝑟 = 0

1 𝜕 2 𝜕𝑅 2𝜇
(𝑟 ) + 2 𝑟 2 (𝐸 − 𝑉(𝑟)) = 0
𝑅(𝑟) 𝜕𝑟 𝜕𝑟 ℏ

1 𝜕 2 𝜕𝑅 2𝜇 𝑘𝑒 2
(𝑟 ) + 2 𝑟 2 (𝐸 + )=0
𝑅(𝑟) 𝜕𝑟 𝜕𝑟 ℏ 𝑟

𝜕 𝜕𝑅 2𝜇 𝑘𝑒 2
𝜕𝑟
(𝑟 2 𝜕𝑟 ) + ℏ2 𝑟 2 (𝐸 + 𝑟
) 𝑅(𝑟) = 0 10.3

Again not an easy differential equation to solve: Let’s try a negative exponential. Negative so
that the probably goes down with large r.

𝑅(𝑟) = 𝐶𝑒 −𝑟/𝑎0
112
𝜕 2 −1 2𝜇 𝑘𝑒 2
(𝑟 ( ) 𝑅(𝑟)) + 2 𝑟 2 (𝐸 + ) 𝑅(𝑟) = 0
𝜕𝑟 𝑎0 ℏ 𝑟

1 2
1 2𝜇 2 𝑘𝑒 2
(−2𝑟 ( ) + 𝑟 2 ) 𝑅(𝑟) + 2 𝑟 (𝐸 + ) 𝑅(𝑟) = 0
𝑎0 𝑎0 ℏ 𝑟

2 1 2𝜇 2𝜇𝑘𝑒 2
(− ( ) 𝑟 + 2 𝑟 2 ) 𝑅(𝑟) + ( 2 𝐸𝑟 2 + 𝑟) 𝑅(𝑟) = 0
𝑎0 𝑎0 ℏ ℏ2

ℏ2
𝑎0 =
𝜇𝑘𝑒 2

2
ℏ2 𝜇 𝑘𝑒 2
𝐸1 = − 2𝜇𝑎2 = − 2 ( ) = 13.6𝑒𝑉 10.4
0 ℏ

𝜓(𝑟, 𝜃, 𝜙) = 𝐶𝑒 −𝑟/𝑎0

where the constant will come from normalizing the wave function.

1 2
𝜓(𝑟, 𝜃, 𝜙) = √4𝜋 𝑒 −𝑟/𝑎0 10.5
√𝑎03

The first wave function is spherically symmetric with a distribution that falls off exponentially
characterized by a radius 𝑎0 , which is identical to the Bohr radius!

Looking at the wave functions above it is clear that the full set of wave functions can be more
complex than spherically symmetric though they will have phi symmetry. The solutions are
called orbitals where the distributions are analogs of orbits with a characteristic vector z about
which there is phi orbital symmetry but otherwise complex functional dependences in theta.
113
The radial wave function, though independent of phi and theta will have complex functional
dependences as well.

We explored the simplest solution to the problem, which corresponds to the ground state of all
three quantum numbers. Both the theta and radial wave equations clearly have more complex
solutions. Also, since each separate wave equation is related by constants, which are different
depending on the quantum numbers, there will be relationships between the three quantum
numbers of the system.

𝜓𝑛𝑙𝑚 (𝑟, 𝜃, 𝜙) = 𝐶𝑛𝑙𝑚 𝑅𝑛𝑙 (𝑟)𝑓𝑙𝑚 (𝜃)𝑔𝑚 (𝜙)

𝑔𝑚 (𝜙) = 𝑒 𝑖𝑚𝜙

(𝑠𝑖𝑛𝜃)|𝑚| 𝑑 𝑙+|𝑚|
𝑓𝑙𝑚 (𝜃) = [𝑑(𝑐𝑜𝑠𝜃)] (𝑐𝑜𝑠 2 𝜃 − 1)𝑙 10.6
2𝑙 𝑙!

𝑚 = −𝑙 … 0 … + 𝑙

Where the f functions are known as the Legendre functions

The combination of f and g functions are often expressed as spherical harmonics

𝑌𝑙𝑚 (𝜃, 𝜙) = 𝐶𝑙𝑚 𝑓𝑙𝑚 (𝜃)𝑔𝑚 (𝜙) 10.7


l=0, m=0
1
𝑌00 (𝜃, 𝜙) = √4𝜋

l=1, m=1,0,-1

114
3
𝑌11 (𝜃, 𝜙) = −√8𝜋 𝑠𝑖𝑛𝜃𝑒 𝑖𝜙

3
𝑌10 (𝜃, 𝜙) = √4𝜋 𝑐𝑜𝑠𝜃

3
𝑌1−1 (𝜃, 𝜙) = √8𝜋 𝑠𝑖𝑛𝜃𝑒 −𝑖𝜙

We have normalized the wave functions over the angular space. The l=0, m=0 state is the one
we already solved.

C𝑟 = 𝑙(𝑙 + 1) connecting the radial and angular equations and quantum numbers.
Note that this is a constant factor with no dependence on the angular or radial coordinates.
However, it is a different constant for each value of l each of which is a separate solution the
angular portion of the Schrodinger equation. For each value of l, and thus different constants,
the solutions to the radial equation will be different.

𝑅𝑛𝑙 (𝑟) = 𝐶𝑛𝑙 𝑒 −𝑟/𝑛𝑎0 𝑟 𝑙 ℒ𝑛𝑙 (𝑟/𝑎0 ) 10.8

n= 1,2,3 …
l= 0, …, n-1

Where the L functions are known as the Laguerre polynomials.

n=1, l=0

2
𝑅10 (𝑟) = 𝑒 −𝑟/𝑎0
√𝑎03

n=2, l=0,1

115
1 𝑟
𝑅20 (𝑟) = (1 − ) 𝑒 −𝑟/2𝑎0
√2𝑎03 2𝑎0

1 𝑟
𝑅21 (𝑟) = 𝑒 −𝑟/2𝑎0
2√6𝑎03 𝑎0

and
2
ℏ2 𝜇 𝑘𝑒 2 𝐸
𝐸𝑛 = −
2𝜇𝑛2 𝑎02
=−
2𝑛2
( ℏ
) = − 𝑛12 10.9

10.2 QUANTUM NUMBERS


The three quantum numbers obtained from solving Equation 10.2 are
n Principal quantum number
l Orbital angular momentum quantum number
ml Magnetic quantum number
Their values are obtained by applying the boundary conditions to the wave function 𝜓(𝑟, 𝜃, 𝜙).
The boundary conditions require that the wave functions have acceptable properties, including
being single valued and finite. The restrictions imposed by the boundary conditions are

These three quantum numbers must be integers. The orbital angular momentum quantum number
must be less than the principal quantum number, l < n, and the magnitude of the magnetic
quantum number (which may be positive or negative) must be less than or equal to the orbital
angular momentum quantum number,|𝑚𝑙 | ≤ 𝑙

10.2.1 The principal quantum number


116
The principal quantum number, n, defines the main energy level (also called electronic shell)
and the size of the orbital. It is assigned the integer values: 1, 2, 3,...etc. As n increases, the
energy and size of the orbital also increase. Upper case letters, K (n = 1), L (n = 2), M (n = 3, N
(n = 4), etc., are sometimes used to denote the electronic shells.

10.2.2 The angular momentum quantum number


The angular momentum quantum number, l, represents the subshells or sublevels within a given
principal shell. IT is also called is the orbital quantum number. It describes the shape of the
orbital. Within a given shell, the orbital quantum number may be assigned any value from 0 to
(n - 1). The sublevels and orbital also use the lower case letters for the symbols, which are: s, p,
d, and f,..., corresponding to l = 0, 1, 2, and 3, respectively.

For example, the notation 3s, 3p, and 3d, describe three different sets of orbital within the
third shell (n = 3). Each of these sets contains orbital with shapes that are different from those
found in other subshells. The quantum notation 1s, 2s and 3s correspond to three sets of orbital in
different shells that contain orbital that are similar in shape, but different in sizes and energy. All
s orbital have spherical shape, but the size increases as the principal quantum number n
increases; that is 1s <2s < 3s.

Each p-subshell contains three orbital, px, py, and pz, each of which has two identical
lobes, like a dumb-bell. All p orbital have the same shape - the dumb-bell shape, with a nodal
plane running through the nucleus, but they have different sizes depending on which shells
(principal energy levels) they are located; thus, 4p > 3p > 2p, (there is no 1p orbital). Electrons
may occupy any one of these orbital, but the probability or electron density decreases and
approaches zero at regions near the nucleus.

10.2.3 The magnetic quantum number


The magnetic quantum number, ml, describes the relative orientation of the orbital in the x-, y-,
and z-coordinates. For each subshell, ml takes values = 0, ±1, ±2, ±3,..,+l. Thus, for each l
117
value there are (2l + 1) values for ml. It also describes the total number of orbital in each subshell
defined by the value l. For example, when n = 2, and l = 0, ml = 0, implying that the 2s subshell
has one orbital only. But, when n = 2, and l = 1, ml has three possible values, -1, 0, and +1,
which implies that there are three p-orbital in the 2p sublevel. (Other p-subshells also have three
orbital per set).

10.2.4 Orbital Shapes and Energy


Although probability has no external boundary, an orbital is assigned an arbitrary
boundary such that within this boundary, the chances of finding an electron with a particular
energy value is greater than 90%. The size and shape of the orbital is determined by the quantum
numbers n and l.

• All orbital with l = 0 such as 1s, 2s, and 3s are spherical, but their sizes increase as the
principal quantum number n increases. Unlike the 1s orbital, the 2s and 3s orbital contain
regions of high probability separated by areas of zero probability called nodes. For s
orbital, the number of radial nodes is (n – 1). Therefore, 1s orbital has no radial nodes, 2s
has one radial node, 3s has 2 radial nodes, etc.

• There are three p-orbital in each sublevel with l = 1. Each p-orbital has two lobes – like a
dumb-bell shape - and a nodal plane through the nucleus. For the p orbital, the number of
nodal planes = l and the number of radial nodes = (n – l – 1). Thus, the 2p orbital has one
nodal plane each, but no radial node; the 3p orbital has one nodal plane and one radial
node; while the 4p orbital has one nodal plane and two radial nodes.

• The d orbital first occur in level n = 3; there is no 1d or 2d. The five 3d-orbitals are 3dxz,
3dxy, 3dyz, 3dx2 – y2, and 3dz2. Each of the four 3d-orbitals (3dxz, 3dxy, 3dyz, and 3dx2 – y2) has
four lobes, with two nodal planes passing through the nucleus. The lobes for 3dxz, 3dxy,
and 3dyz lie on the planes between the respective axes; while the four lobes in the 3dx2 – y2
orbital lie along the xy-axes. The 4d and 5d orbital also look like the 3d but have larger
lobes.
118
• The f orbital first occurs is level n = 4, and their shapes are more complex.

In the hydrogen atom, the energy of a particular orbital is determined only by the principal
quantum number n. Thus, all orbital in the same principal energy level, such as 3s, 3p, and 3d, all
have the same energy – they are said to be degenerate. The one electron in hydrogen atom may
occupy any of its atomic orbital, but when in the lowest energy state, called the ground state, this
electron resides in the 1s orbital. If the atom absorbs sufficient amount of energy, the electron
may be excited to a higher-energy orbital, producing an excited state.

What is the degeneracy of the n = 3 level? That is, how many different states are contained in the
energy level E3 = - E0/9?

To find the total degeneracy for n = 3, we have to add all the possibilities.

The n = 3 level is degenerate (in the absence of a magnetic field) because all nine states have the
same energy but different quantum numbers.

A set of three quantum numbers, n, l, and ml, are required to describe any orbital in an
atom. However, a fourth quantum number, called the spin quantum number, ms, is needed to
describe a particular electron in an atom. ms can only have one of the two possible values, which

119
is ½ or –½, which indicates the direction of electron spin. This fourth quantum number is the
result of observations that a beam of atoms containing odd number of electrons is split into two
after passing through a strong magnetic field. The details of this will be discussed in the next
study session

The significance of electron spins is associated with the Pauli Exclusion Principle,
which states that, in a given atom no two electrons can have the same set of four quantum
numbers (n, l, ml, and ms). This postulate suggests that if two electrons occupy the same orbital,
they must have opposite spins, that is, different values of ms. As a consequence of the Pauli
Exclusion principle, an orbital may accommodate a maximum number of two electrons.

SUMMARY
The Schrodinger wave theory is applied to atomic physics, beginning with the hydrogen atom.
The application of the boundary conditions leads to three quantum numbers:
n Principal quantum number
l Orbital angular momentum quantum number
ml Magnetic quantum number
The principal quantum number, n, defines the main energy level (also called electronic shell)
and the size of the orbital.
The angular momentum quantum number, l, (also called the orbital quantum number), represents
the subshells or sublevels within a given principal shell.
The magnetic quantum number, ml, describes the relative orientation of the orbital in the x-, y-,
and z-coordinates.

SELF ASSESSMENT QUESTIONS


1. A hydrogen atom is in a state for which the principle quantum number is n = 3. How
many possible such states are there for which the magnetic quantum number is m = 0 ?

120
(a) 2
(b) 6
(c) 10
(d) 4 \
2. An electron in a hydrogen atom is described by the quantum numbers: n = 8 and m = 4 .
What are the possible values for the orbital quantum number ?
(a) only 0 or 4
(b) only 4, 5, 6, or 7
(c) only 4 or 7
(d) only 5, 6, 7, or 8
3. Determine the maximum number of electron states with principal quantum number n
= 3?
(a) 2
(b) 6
(c) 18
(d) 3
4. Which one of the following values of m is not possible for = 2?
(a) zero
(b) +1
(c) +3
(d) –1
5 An electron in an atom has the following set of quantum numbers:
n = 3, = 2, m = +1, ms = +1/2.
6. What shell is this electron occupying?
(a) K shell (b) M shell (c) O shell
(d) L shell
7. In which subshell can the electron be found?
(a) s (b) d (c) g
(b) p

121
8. According to the quantum mechanical picture of the atom, which quantum number(s)
could be different for electrons in this same atom that have exactly the same energy?
(a) n, , m and ms
(b) only and m

(c) ms
(d) only , m and ms

9. Use all four quantum numbers to write down all possible sets of quantum numbers for the
4f state of atomic hydrogen. What is the total degeneracy?

10. Use all four quantum numbers to write down all possible sets of quantum numbers for the
5d state of atomic hydrogen. What is the total degeneracy?

REFERENCES
Stephen T. Thornton and Andrew Rex (2013), Modern physics for scientists and Engineers, 4th
Edition, Charles Hartford Publishers.

Hugh D. Young and Roger A. Freedman, (2004), University Physics with Modern Physics,11th
Edition, Pearso n Education Publishers

Arthur Beiser (2003), Concepts of Modern Physics, 6th Edition, McGraw-Hill Publishers

Should you require more explanation on this study session, please do not hesitate to contact your

e-tutor via the LMS.

Are you in need of General Help as regards your studies? Do not hesitate to contact
the DLI IAG Center by e-mail or phone on:

iag@dli.unilag.edu.ng
122

08033366677
STUDY SESSION 11: ELECTRON SPIN

INTRODUCTION
Quantum mechanics explains certain properties of the hydrogen atom in a way that is more
precise and accurate. However, we have to take into account electron spin and the exclusion
principle. In this study session we shall consider the role of electron spin in atomic phenomena
and the exclusion principle is the key to understanding the structures of atoms with more than
one electron.

LEARNING OUTCOMES
At the end of the lesson, the student should be able to:
11.1 Explain the concept of electron spin
11.2 State the Pauli’s Exclusion Principle
11.3 State the expression for the symmetric and Antisymmetric wave function
11.4 Identify the relationship between the Pauli’s Principle and the fermions and Bosons

11.1 ELECTRON SPIN


It is imperative to know that the theory of the atom developed in the previous study session
cannot account for some well-known experimental observations. For instance the many spectral
lines actually consist of two separate lines that are very close together. This can be observed in
the fine structure is the first line of the Balmer series of hydrogen, which arises from transitions
between the n = 3 and n = 2 levels in hydrogen atoms. Here the theoretical prediction is for a
single line of wavelength 656.3 nm while in reality there are two lines 0.14 nm apart—a small
effect, but a conspicuous failure for the theory.

123
Another failure of the simple quantum-mechanical theory of the atom occurs in the Zeeman
effect which was just discussed in the last study session. There we saw that the spectral lines of
an atom in a magnetic field should each be split into the three components specified by Equation
9.24. While the normal Zeeman effect is indeed observed in the spectra of a few elements under
certain circumstances, more often it is not. Four, six, or even more components may appear, and
even when three components are present their spacing may not agree with Equation 9.24. Several
anomalous Zeeman patterns are shown in Figure 11.1 together with the predictions of Equation
9.24.

Figure 11.1: The normal and anomalous Zeeman effects in various spectral lines

In order to account for both fine structure in spectral lines and the anomalous Zeeman effect, two
Dutch graduate students, Samuel Goudsmit and George Uhlenbeck, proposed in 1925 that:

Every electron has an intrinsic angular momentum, called spin, whose


magnitude is the same for all electrons. Associated with this angular
momentum is a magnetic moment.

Goudsmit and Uhlenbeck thought of a classical picture of an electron as a charged sphere


spinning on its axis. The rotation involves angular momentum, and because the electron is
124
negatively charged, it has a magnetic moment µs opposite in direction to its angular momentum
vector S. The concept of electron spin proved to be successful in explaining not only fine
structure and the anomalous Zeeman effect but a wide variety of other atomic effects as well. To
be sure, the picture of an electron as a spinning charged sphere is open to serious objections. For
one thing, observations of the scattering of electrons by other electrons at high energy indicate
that the electron must be less than 10-16m across, and quite possibly is a point particle. In order to
have the observed angular momentum associated with electron spin, so small an object would
have to rotate with an equatorial velocity many times greater than the velocity of light.

In 1929, the fundamental nature of electron spin was confirmed by Paul Dirac’s development of
relativistic quantum mechanics. He found that a particle with the mass and charge of the electron
must have the intrinsic angular momentum and magnetic moment proposed for the electron by
Goudsmit and Uhlenbeck.

The quantum number s describes the spin angular momentum of the electron. The only value s
1
can have is s = 2, which follows both from Dirac’s theory and from spectral data. The magnitude

S of the angular momentum due to electron spin is given in terms of the spin quantum number s
by
√3
𝑆 = √𝑠(𝑠 + 1) ℏ = ℏ 11.1
2

This is the same formula as that giving the magnitude L of the orbital angular momentum in
terms of the orbital quantum number l.
𝐿 = √𝑙(𝑙 + 1) ℏ 11.2

Find the equatorial velocity v of an electron under the assumption that it is a uniform sphere of
radius r = 5.00 x 10-17m that is rotating about an axis through its center.

125
2
The angular momentum of a spinning sphere is Iω, where 𝐼 = 𝑚𝑟 2 is its moment of inertia
5

and 𝜔 = 𝑣/𝑟 is its angular velocity. From Equation 9.1, the spin angular momentum of an
electron is
√3 2 𝑣 2
𝑆= ℏ = 𝐼𝜔 = (5 𝑚𝑟 2 ) (𝑟 ) = 𝑚𝑣𝑟
2 5

5√3 ℏ 5√3(1.055 × 10−34 )


𝑣= ( )= = 5.01 × 1012 𝑚/𝑠 = 1.67 × 104 𝑐
4 𝑚𝑟 4(9.11 × 10−31 )(5 × 10−17)

Thus, the equatorial velocity of an electron on the basis of this model must be over 10,000 times
the velocity of light, which is impossible. No classical model of the electron can overcome this
difficulty.
The space quantization of electron spin is described by the spin magnetic quantum number ms.
We recall that the orbital angular-momentum vector can have the 2l + 1 orientations in a
magnetic field from +l to -l. Similarly the spin angular-momentum vector can have the 2s + 1 = 2
1 1
orientations specified by 𝑚𝑠 = + (spin up) and 𝑚𝑠 = − (spin down) as shown in figure 9.2.
2 2

126
Figure 11.2 The two possible orientations of the spin angular momentum vector are the
“spin up” and “spin down”

The component Sz of the spin angular momentum of an electron along a magnetic field in the z
direction is determined by the spin magnetic quantum number, so that
1
𝑆𝑧 = 𝑚𝑠 ℏ = ± 2 ℏ 11.3

Recall that the gyromagnetic ratio is the ratio between magnetic moment and angular
momentum. The gyromagnetic ratio for electron orbital motion is –e/2m. The gyromagnetic ratio
characteristic of electron spin is almost exactly twice that characteristic of electron orbital
motion. Taking this ratio as equal to 2, the spin magnetic moment µs of an electron is related to
its spin angular momentum S by
𝑒
𝜇𝑠 = − 𝑚 𝑆 11.4

The possible components of 𝜇𝑠 along any axis, say the y axis, are therefore limited to
𝑒ℏ
𝜇𝑠𝑦 = ± 2𝑚 = ±𝜇𝐵 11.5

Where 𝜇𝐵 is the Bohr magneton which is 9.274 x 10-24 J/T = 5.788 x 10-5eV/T).
The introduction of electron spin into the theory of the atom means that a total of
four quantum numbers, n, l, ml, and ms, is needed to describe each possible state of
an atomic electron. These are listed in Table 9.1.

127
Table 11.1 Quantum numbers of an atomic electron

11.2 EXCLUSION PRINCIPLE


In 1925 Wolfgang Pauli discovered the fundamental principle that governs the electronic
configurations of atoms having more than one electron. His exclusion principle states that
No two electrons in an atom can exist in the same quantum state. Each electron must have
a different set of quantum numbers n, l, ml, ms.
Pauli was led to the exclusion principle by a study of atomic spectra. The various states of an
atom can be determined from its spectrum, and the quantum numbers of these states can be
inferred. In the spectra of every element but hydrogen a number of lines are missing that
correspond to transitions to and from states having certain combinations of quantum numbers.

To illustrate the exclusion principle and some of the evidence for it, let us consider two simple
atoms, helium and lithium. First, let us imagine putting together a helium atom (Z = 2) from a
helium nucleus and two electrons. If we add one electron to the nucleus, its lowest possible state
is the 1s state (n = 1, l = m = 0) with its spin either up or down (ms = ± 1
2 ). If we next add the
second electron, it too can go into the 1s state. But according to the exclusion principle the two
electrons cannot occupy exactly the same quantum state. Since they have the same values of n, l,
and m, they must have different values of ms; that is, if both electrons are in the 1s state, their
spins must be antiparallel.

128
The situation with the excited states of helium is different. For example, there is an excited state
with one electron in the 1s level and the other in the 2s level. In this case the two electrons are
certainly in different quantum states, whatever their spin orientations (parallel or antiparallel).
Thus the Pauli principle does not forbid either of these arrangements, and both are observed, the
first with   0 and the second with  = 0. As a second example, let us imagine putting together
a lithium atom (Z = 3) from a lithium nucleus and three electrons. When we add the first two
electrons they can both go into the 1s level, provided that their spins are antiparallel. But since
there are only two possible orientations of the spin, there is now no way in which the third
electron can go into the 1s level. The Pauli principle requires that the third electron go into some
higher level, the lowest of which is the 2s.

In general, the Pauli principle implies that any s level (1s, 2s, etc.) can accommodate two
electrons but no more. Levels with higher angular momentum can accommodate more electrons,
because their degeneracy is larger. For example, any level with l = 1 contains six distinct
quantum states (6 = 2  3, since there are two orientations of S and three orientations of L);
therefore, any p level (l = 1) can accommodate six electrons, but no more. Similarly, any d level,
with l = 2, has ten distinct states (2  5) and can accommodate ten electrons, but no more.

11.3 SYMMETRIC AND ANTISYMMETRIC WAVE FUNCTIONS


Before we explore the role of the exclusion principle in determining atomic structures, it is
interesting to look into its quantum-mechanical implications.
The complete wave function ψ(1, 2, 3, . . . , n) of a system of n non-interacting particles can be
expressed as the product of the wave functions ψ(1), ψ(2), ψ (3), . . . ,ψ(n) of the individual
particles. That is,
ψ (1, 2, 3, . . . , n) = ψ (1) ψ (2) ψ (3) . . . ψ (n) 11.6

Now let’s use equation 9.6 to study the kinds of wave functions that can be used to describes a
system of two identical particles.

129
Suppose one of the particles is in quantum state a and the other in state b. Because the particles
are identical, it should make no difference in the probability density |𝜓|2 of the system if the
particles are exchanged, with the one in state a replacing the one in state b, and vice versa.

Symbolically, we require that |𝜓|2 (1, 2) =|𝜓|2 (2, 1) 11.7

The wave function 𝜓(2, 1) that represents the exchanged particles can be either

Symmetric 𝜓(2, 1) = 𝜓 (1, 2) 11.8


or
Antisymmetric 𝜓(2, 1) = −𝜓 (1, 2) 11.9

and still fulfill Equation 9.7. The wave function of the system is not itself a measurable quantity,
and so it can be altered in sign by the exchange of the particles. Wave functions that are
unaffected by an exchange of particles are said to be symmetric, while those that reverse sign
upon such an exchange are said to be antisymmetric.
If particle 1 is in state a and particle 2 is in state b, the wave function of the system
is, according to Equation 9.6,
𝜓1 = 𝜓𝑎 (1)𝜓𝑏 (2) 11.10

If particle 2 is in state a and particle 1 is in state b, the wave function is


𝜓2 = 𝜓𝑎 (2)𝜓𝑏 (1) 11.11

Because the two particles are indistinguishable, we have no way to know at any moment
Whether 𝜓1 or 𝜓2 describes the system. The likelihood that 𝜓1 is correct at any moment is the
same as the likelihood that 𝜓2 is correct.
Equivalently, we can say that the system spends half the time in the configuration whose wave
function is 𝜓1 and the other half in the configuration whose wave function is 𝜓2 . Therefore a

130
linear combination of 𝜓1 and 𝜓2 is the proper description of the system. Two such combinations,
symmetric and antisymmetric, are possible:

Symmetric 11.12

Antisymmetric 11.13

The factor 1⁄√2 is needed to normalize 𝜓S and 𝜓A. Exchanging particles 1 and 2 leaves 𝜓S
unaffected, while it reverses the sign of 𝜓A. Both 𝜓S and 𝜓A obey Equation 9.7.

There are a number of important distinctions between the behavior of particles in systems whose
wave functions are symmetric and that of particles in systems whose wave functions are
antisymmetric. The most obvious is that in the symmetric case, both particles 1 and 2 can
simultaneously exist in the same state, with a = b. In the antisymmetric case, if we set a = b, we
find that

11.14

Hence the two particles cannot be in the same quantum state. Pauli found that no two electrons in
an atom can be in the same quantum state, so we conclude that systems of electrons are described
by wave functions that reverse sign upon the exchange of any pair of them.

11.4 FERMIONS AND BOSONS; THE ORIGIN OF THE PAULI PRINCIPLE


In this section we describe how the Pauli principle follows from certain symmetry properties of
the multiparticle wave function. Before exploring further the consequences of the Pauli
principle, we take a moment to describe in this section where the principle comes from. One can
take the view that the Pauli principle is an observed property of electrons (and some other
particles, including protons and neutrons) — a property for which there is overwhelming
experimental evidence. If you would like to take this view, then you can safely skip this section
131
for now. Nevertheless, the Paul principle actually follows from a more fundamental idea — the
complete indistinguishability of identical particles in quantum mechanics. This result is
interesting in its own right and has a remarkable consequence. All the particles of nature fall into
just two categories: First there are the so-called fermions, which do obey the Pauli principle, and
second there are the bosons which do not.

SUMMARY
Every electron has an intrinsic angular momentum, called spin, whose magnitude is the same for
all electrons. Associated with this angular momentum is a magnetic moment.

The magnitude S of the angular momentum due to electron spin is given in terms of the spin
quantum number s by
√3
𝑆 = √𝑠(𝑠 + 1) ℏ = ℏ
2

This is the same formula as that giving the magnitude L of the orbital angular momentum in
terms of the orbital quantum number l.
𝐿 = √𝑙(𝑙 + 1) ℏ

The spin magnetic moment µs of an electron is related to its spin angular momentum S by
𝑒
𝜇𝑠 = − 𝑚 𝑆

The possible components of 𝜇𝑠 along any axis, say the y axis, are therefore limited to
𝑒ℏ
𝜇𝑠𝑦 = ± = ±𝜇𝐵
2𝑚

The Exclusion Principle states that: No two electrons in an atom can exist in the same quantum
state. Each electron must have a different set of quantum numbers n, l, ml, ms.

132
Wave functions that are unaffected by an exchange of particles are said to be symmetric, while
those that reverse sign upon such an exchange are said to be antisymmetric.

All the particles of nature fall into just two categories: First there are the so-called fermions,
which do obey the Pauli principle, and second there are the bosons which do not.

SELF ASSESSMENT QUESTIONS

(1) Which of the following statements best describes the quantum property spin?
a) Spin is a measure of the rotation rate of a subatomic particle.
b) Spin is a measure of the rate at which a particle spins around (orbits) another particle.
c) Spin is a property that applies only to large objects, like baseballs.
d) Spin is not meant to be taken literally but measures the inherent angular momentum of a
subatomic particle.

(2) The characteristic that distinguishes fermions from bosons is


a) their mass.
b) their electric charge.
c) their spin.
d) their size.

(3) Which of the following is not a valid set of quantum numbers?

a) n = 2, l = 1, ml = 0, and ms = -1/2
b) n = 2, l = 1, ml = -1, and ms = -1/2
c) n = 3, l = 0, ml = 0, and ms = 1/2
d) n = 3, l = 2, ml = 3, and ms = 1/2

133
(4) An electron in a 4p orbital can have a wave function with which of the following set of
quantum numbers, (n, l, ml, ms)?
a) (4, 0, 0, 1/2)
b) (4, 1, –1, –1/2)
c) (5, 4, 1, –1/2)
d) (5, 4, 4, 1/2)

(5) Based on their spin, all particles fall into which of the following categories?
a) quarks and leptons
b) fermions and bosons
c) matter and antimatter
d) color, flavor, and mass

(6) Which one of the following factors best explains why the six electrons of a carbon
atom are not all in the 1s state?
(a) electron spin
(b) Heisenberg uncertainty principle
(c) Rutherford model of atomic structure
(d) Pauli exclusion principle

(7) Which one of the following statements concerning the electrons specified by the notation
3p4 is true?
(a) The electrons are in the M shell.
(b) The electrons are in the = 2 subshell.
(c) The electrons are necessarily in an excited state.
(d) They have principal quantum number 4.

(8) How many electrons could be accommodated in a g subshell?


(a) 4

134
(b) 8
(c) 18
(d) 5

(9) Which one of the following electronic configurations corresponds to an atomic


ground state?
(a) 1s2 2s1 2p6
(b) 1s1 2s2 3p1
(c) 1s1 2s1 2p1
(d) 1s2 2s2 2p1

(10) State the Pauli’s Exclusion Principle

REFERENCES

Hugh D. Young and Roger A. Freedman, (2004), University Physics with Modern Physics,11 th
Edition, Pearson Education Publishers

Arthur Beiser (2003), Concepts of Modern Physics, 6th Edition, McGraw-Hill Publishers

Should you require more explanation on this study session, please do not hesitate to contact your

e-tutor via the LMS.

Are you in need of General Help as regards your studies? Do not hesitate to contact
the DLI IAG Center by e-mail or phone on:

iag@dli.unilag.edu.ng
08033366677
135
STUDY SESSION 12: PERIODIC TABLE

INTRODUCTION
The periodic table, also known as Mendeleev's table, is a table of the chemical elements existing
on Earth. The Russian chemist Dmitri Mendeleev is credited with its creation in 1869, although
less extensive tables existed before this. He intended to display the patterns apparent in the
chemical properties of each element. Since its original creation, new elements have been
discovered and added to Mendeleev's initial table.

LEARNING OUTCOMES
At the end of this study session, the student should be able to;
12.1 Explain the development of the periodic table
12.2 Highlight and explain briefly the atomic properties

12.1 THE PERIODIC TABLE


By the middle of nineteenth century, chemists had discovered a large number of elements,
determined their relative atomic masses, and measured a host of their properties. Now they
needed a system of classification that would enable them to put these elements into various

136
groups based on their physical and chemical properties. They searched for patterns among the
various properties of elements already known.

One of them, Johann Dobereiner (1780-1849), introduced the so called Dobereiner’s triad. That
is, he placed elements in groups of three, based on their atomic masses, such that the atomic
mass of the middle element appeared to be approximately equal to the average mass of the other
two elements (with lower and higher atomic masses). For example, the atomic mass of bromine
(81) is approximately the average of those of chlorine (35) and iodine (127). (The actual atomic
mass for bromine is 79.9) However, the “triad model” was found to be applicable only to certain
groups of elements. In 1864, an English chemist named John Newlands proposed that elements
should be arranged in octaves, like the musical scale.
In 1869 the Russian chemist Dmitri Mendeleev formulated the periodic law whose modern
statement is

When the elements are listed in order of atomic number, elements


with similar chemical and physical properties recur at regular intervals.

Although the modern quantum theory of the atom was many years in the future, Mendeleev was
fully aware of the significance his work would turn out to have. A periodic table is an
arrangement of the elements according to atomic number in a series of rows such that elements
with similar properties form vertical columns. Elements with similar properties form the groups
shown as vertical columns in Table 12.1

137
Table 12.1: Periodic Table

Table 12.1 is a simple form of periodic table. Thus group 1 consists of hydrogen plus the alkali
metals, which are all soft, have low melting points, and are very active chemically. Lithium,
sodium, and potassium are examples. Hydrogen, although physically a nonmetal, behaves
chemically much like an active metal. Group 7 consists of the halogens, volatile nonmetals that
form diatomic molecules in the gaseous state. Like the alkali metals, the halogens are chemically
active, but as oxidizing agents rather than as reducing agents. Fluorine, chlorine, bromine, and
iodine are examples; fluorine is so active it can corrode platinum. Group 8 consists of the inert
gases, of which helium, neon, and argon are examples. As their name suggests, they are inactive
chemically: they form virtually no compounds with other elements, and their atoms do not join
together into molecules. The horizontal rows in Table 12.1 are called periods. The first three
periods are broken in order to keep their members aligned with the most closely related elements
of the long periods below.

138
Figure 12.2: The majority of the elements are metals

Most of the elements are metals (Figure 12.2). Across each period is a more or less steady
transition from an active metal through less active metals and weakly active nonmetals to highly
active nonmetals and finally to an inert gas. In each column there are also regular changes in
properties, but they are far less obvious than those in each period. For example, increasing
atomic number in the alkali metals is accompanied by greater chemical activity, while the
reverse is true in the halogens.

A series of transition elements appears in each period after the third between the group 2 and
group 3 elements. The transition elements are metals, in general hard and brittle with high
melting points that have similar chemical behavior. Fifteen of the transition elements in period 6
are virtually indistinguishable in their properties and are known as the lanthanide elements (or
rare earths). Another group of closely related metals, the actinide elements, is found in period
7. For over a century the periodic law has been indispensable to chemists because it provides a
framework for organizing their knowledge of the elements. It is one of the triumphs of the
quantum theory of the atom that it enables us to account in a natural way for the periodic law
without invoking any new assumptions.

12.2 ATOMIC PROPERTIES


The following features are used to determine the structure of atoms.

12.2.1 Atomic radius

139
Since orbital have no specific boundaries, the size of an atom cannot be specified
precisely. The easiest way to express the size of an atom is through the measurement of its
radius, and there are two ways in which this can be accomplished. For elements that form
diatomic molecules, the atomic radius is equal to one-half the inter-nuclear distance. This is
called the covalent atomic radius. For example, X-ray crystallography determined that the
internuclear distance in Br2 is 228 pm. The covalent atomic radius for bromine is 114 pm.

For nonmetals that do not form diatomic molecules, the atomic radii are estimated from
their various covalent compounds. The radii for metals atoms (called metallic radii) are obtained
from half the internuclear distance of two adjacent atoms in metal crystals. Covalent atomic radii
are always smaller than if the radii were estimated from the 90% electron density volumes of
isolated atoms. This is because, when two atoms approach each other to form covalent bonds,
their “electron cloud” interpenetrates. General, the atomic radii decrease in going from left to
right across a given period in the periodic table.

This decrease in atomic radii is due to the increasing “effective nuclear charge” in going from
left to right, and valence electrons are drawn closer to the nucleus, decreasing the atomic size.
For example,
Li > Be > B > C > N > O > F;Na > Mg > Al > Si > P > S > Cl;
• Atomic radii increase from top to bottom down a group.
This is because of the increase in orbital size due to increasing principal quantum levels. As one
goes down a given group, the valence electrons occupy energy levels with a higher quantum
number than the one before. For example,

Li < Na < K < Rb < Cs; F < Cl < Br < I


• The radii of cations are smaller and the radii of anions are larger than those of the
corresponding atoms from which the ions are derived.

12.2.2 Ionization Energy

140
Ionization energy is the amount of energy required to remove an electron from a gaseous
atom or ion:
M(g) → M+(g) + e-; M+(g) → M2+(g) + e- ;
Where the atom or ion is assumed to be in its ground state (lowest electronic energy state). The
ionization energy values are normally expressed in kJ/mol. For example,

Al(g) → Al+(g) + e- ; I1 = 580 kJ/mol;


Al+(g) → Al2+(g) + e- ; I2 = 1815 kJ/mol;
Al2+(g) → Al3+(g) + e- ; I3 = 2740 kJ/mol;
Al3+(g) → Al4+(g) + e- ; I4 = 11, 600 kJ/mol;
The first ionization energy (I1) is the energy needed to remove an electron from a neutral
atom, and this would be the electron occupying the highest energy orbital. For aluminum
([Ne]3s23p1), it will be the electron in the 3p orbital, and its removal yields Al+ ion with electron
configuration [Ne]3s2. The next two electrons are removed from the 3s orbital, and the fourth
from the 2p orbital. The removal of second electron requires about three times as much energy as
the first one. This is because, the electrons are at a lower electronic energy state (more stable
state) and after the removal of the first electron, the effective nuclear charge on the remaining
electrons increases and the removal of the subsequent electrons becomes more difficult.

12.2.3 Electron Affinity


Electron affinity is the energy change associated with the addition of an electron to a
gaseous atom:
X(g) + e- → X-(g)

Electron affinities generally have negative values because the process is exothermic – the added
electrons are experiencing a net nuclear attractions when entering the atoms.

Electron affinity generally increases (become more negative or more exothermic) from
left to right across periods and decreases (become less negative) top to bottom down groups,
although certain anomalies are noted. Both nuclear attractions and electron repulsions appear to
141
influence electron affinity. For example, nitrogen atom does not form stable, isolated N-(g) ion,
whereas its neighbors, carbon and oxygen, form stable C-(g) and O-(g) ions. These facts reflect
the difference in their electron configurations. Nitrogen has the configuration 1s2 2s2 2p3, in
which the 2p orbital are half-filled (a stable configuration). Adding one more electron to the 2p
orbital would decrease the electronic stability due to increasing electron repulsions. Carbon has
the configuration [He]2s22p2, where one of the 2p orbital is empty. An electron added to carbon
would occupy this empty 2p orbital, which would not result in a significant electron repulsion.
While oxygen contains one more proton than nitrogen; although adding an electron to oxygen
would increase the electron repulsion, this will be countered by the higher nuclear attraction in
oxygen.

12.2.4 The Alkali Metals


Representative elements within the same group exhibit similar chemical properties that
change in a regular pattern. This is explained by the fact that these elements have similar
valence-shell electron configuration. It is the electron configuration of the valence shell that
primarily determines the chemical properties of an atom.

The alkali metals (Group 1A: Li, Na, K, Rb, Cs, and Fr) represent the most reactive
group of metals. As a group, they have the largest atomic sizes and lowest ionization energies.
The large shielding effect by “core” (inner-shell) electrons results in weak effective nuclear
charge experienced by the valence electrons. This results in their relatively low ionization energy
and consequently, their high reactivity. The alkali metals have relatively low melting points and
they are generally soft. This is because they have weak metallic bonds that result from the single
valence electrons per atom. Their melting points decrease going down the group. The density of
the alkali metals increases going down the group. This is because atomic masses increase more
rapidly than atomic sizes.

As atomic size increases down the group, ionization energy decreases and reactivity
increases– the lower the ionization energy, the more easily the atoms lose the single valence-
142
shell electrons. Thus, francium would be the most reactive metal and lithium the least reactive in
that group. Alkali metals are strong reducing agents. The expected reducing trend for these
metals is Li < Na < K < Rb < Cs.

SUMMARY
A periodic table is an arrangement of the elements according to atomic number in a series of
rows such that elements with similar properties form vertical columns. Elements with similar
properties represented in columns forms the groups while those represented in the horizontal
sections of the periodic table forms the periods.

Ionization energy is the amount of energy required to remove an electron from a gaseous atom
or ion:
Electron affinity is the energy change associated with the addition of an electron to a gaseous
atom.

SELF ASSESSMENT QUESTIONS


Consider the following list of electron configurations:
(1) 1s2 2s2 3s2 (4) 1s2 2s2 2p6 3s2 3p6 4s2
(2) 1s2 2s2 2p6 (5) 1s2 2s2 2p6 3s2 3p6 4s2 3d6
(3) 1s2 2s2 2p6 3s1

1. Which one of the above lists represents the electronic configuration for the ground
state of the atom with Z = 11?
(a) 1 (b) 3 (c) 5 (d) 2

2. Which electronic configuration is characteristic of noble gases?


(a) 1 (b) 3 (c) 5 (d) 2

143
3. Which one of the above configurations represents a neutral atom that readily forms a
singly charged positive ion?
(a) 1 (b) 3 (c) 5 (d) 4

4. Which one of the above configurations represents an excited state of a neutral atom?
(a) 1 (b) 3 (c) 5 (d) 2

5. Which one of the above configurations represents a transition element?


(a) 1 (b) 3 (c) 5 (d) 4

6. A neutral sulfur atom has how many valence electrons?


(a) 2 (b) 4 (c) 6 (d) 16

7. Briefly explain the development of the periodic table


8. List and explain the various atomic properties

REFERENCES
Hugh D. Young and Roger A. Freedman, (2004), University Physics with Modern Physics,11 th
Edition, Pearson Education Publishers

Arthur Beiser (2003), Concepts of Modern Physics, 6th Edition, McGraw-Hill Publishers

Should you require more explanation on this study session, please do not hesitate to contact your

e-tutor via the LMS.

Are you in need of General Help as regards your studies? Do not hesitate to contact
the DLI IAG Center by e-mail or phone on:

144
iag@dli.unilag.edu.ng
08033366677
STUDY SESSION 13: X-RAY SPECTRA

INTRODUCTION
Electronic transitions within the inner shells of heavier atoms are accompanied by large energy
transfers. If the excess energy is carried off by a photon, x rays are emitted at specific
wavelengths peculiar to the emitting atom. This explains why discrete x-ray lines are produced
when energetic electrons bombard a metal target. Consequently, in this study session, we shall
examine the concept of continuous X-ray spectrum that are formed and various applications of x-
ray spectra.

LEARNING OUTCOMES
At the end of this study session, student should be able to;
13.1 Explain continuous X-ray spectrum
13.2 State Mosley’s law
13.3 State the applications of X-ray spectra

13.1 CONTINUOUS X-RAY SPECTRUM

145
The continuous x-ray spectrum is the result of the inverse photoelectric effect, with electron
kinetic energy being transformed into photon energy. The line spectrum, on the other hand,
comes from electronic transitions within atoms that have been disturbed by the incident
electrons. The inner electrons of heavier elements are of different matter, because these electrons
are not well shielded from the full nuclear charge by intervening electron shells and so are very
tightly bound. In sodium, for example, only 5.13eV is needed to remove the outermost 3s
electron, whereas the corresponding figures for the inner ones are 31eV for each 2p electron, 63
eV for each 2s electron, and 1041eV for each 1s electron. Transitions that involve the inner
electrons in an atom are what give rise to x-ray line spectra because of the high photon energies
involved. Figure 11.1 shows the energy levels (not to scale) of a heavy atom. The energy
differences between angular momentum states within a shell are minor compared with the energy
differences between shells.

146
Figure 13.1: Origin of X-ray spectra

13.2 MOSELEY’S LAW AND ATOMIC ENERGY LEVELS


We may find sharp peaks superimposed on this continuous spectrum depending on the
accelerating voltage and the target element as shown in figure 13.2.

147
Figure 13.2: Graph of intensity per unit wavelength as a function of wavelength for x-
rays produced with an accelerating voltage of 35 kV and a molybdenum target.

These peaks are at different wavelengths for different elements; they form what is called a
characteristic x-ray spectrum for each target element. In 1913 the British scientist Henry G. J.
Moseley studied these spectra in detail using x-ray diffraction techniques. He found that the most
intense short-wavelength line in the characteristic x-ray spectrum from a particular target
element, called the line, varied smoothly with that element’s atomic number. This is in sharp
contrast to optical spectra, in which elements with adjacent have spectra that often bear no
resemblance to each other. Moseley found that the relationship could be expressed in terms of x-
ray frequencies ƒ by a simple formula called Moseley’s law:

𝑚𝑒 4 1 1
𝑓 = 8𝜖2 ℎ3 (𝑛2 − 𝑛2 ) (𝑧 − 1)2 13.1
𝑜 𝑓 𝑖

3𝑐𝑅(𝑍−1)2
𝑓= 13.2
4

Where R is the Rydberg constant


𝑚𝑒 4
𝑅= = 1.097 × 107 𝑚−1
8𝜖𝑜2 𝑐ℎ3
Moseley went far beyond this empirical relationship; he showed how characteristic x-ray spectra
could be understood on the basis of energy levels of atoms in the target. His analysis was based
on the Bohr model. In transitions from excited states to the ground state, they usually emit
photons in or near the visible region. Characteristic x rays, by contrast, are emitted in transitions
involving the inner shells of a complex atom. In an x-ray tube the electrons may strike the target
with enough energy to knock electrons out of the inner shells of the target atoms. These inner
electrons are much closer to the nucleus than are to the electrons in the outer shells; they are
much more tightly bound, and hundreds or thousands of electron volts may be required to
remove them. Suppose one electron is knocked out of the shell. This process leaves a vacancy,
which we’ll call a hole. (One electron remains in the shell.) The hole can then be filled by an
148
electron falling in from one of the outer shells, such as the L, M, N,… shell. This transition is
accompanied by a decrease in the energy of each state and it has definite energy, so the emitted
x-rays have definite wavelengths; the emitted spectrum is a line spectrum.

Estimate the wavelength for an n = 2 to n = 1 transition in molybdenum (Z = 42). What is the


energy of such photon?

From equation 13.1


𝑛𝑓2 = 4
𝑛𝑖2 = 1
Since c = fλ
1 𝑚𝑒 4 1 1
Then 𝜆 = 8𝜖2 𝑐ℎ3 (𝑛2 − 𝑛2 ) (𝑧 − 1)2
𝑜 𝑓 𝑖

1 1 1
= 1.097 × 107 (41) 2 ( − ) = 1.38 × 1010 𝑚−1
𝜆 1 4
1
𝜆= = 0.072𝑛𝑚
1.38 × 1010

ℎ𝑐 (6.63 × 10−34 )(3.00×108 )


Energy = 𝐸 = ℎ𝑓 = = = 17𝑘𝑒𝑉
𝜆 (7.2 ×10−11 )(1.6 × 10−19 )

13.3 X-RAY ABSORPTION SPECTRA


We can also observe x-ray absorption spectra. Unlike optical spectra, the absorption wavelengths
are usually not the same as those for emission, especially in many-electron atoms, and do not
give simple line spectra. For example, the emission line results from a transition from the shell to
a hole in the shell. The reverse transition doesn’t occur in atoms with because in the atom’s
ground state, there is no vacancy in the shell. To be absorbed, a photon must have enough energy
149
to move an electron to an empty state. Since empty states are only a few electron volts in energy
below the free-electron continuum, the minimum absorption energies in many-electron atoms are
about the same as the minimum energies that are needed to remove an electron from its shell.
Experimentally, if we gradually increase the accelerating voltage and hence the maximum
photon energy, we observe sudden increase in absorption when we reach these minimum
energies. These sudden jumps of absorption are called absorption edges (Figure 13.3).

Figure 13.3: Absorption edges

Characteristic x-ray spectra provide a very useful analytical tool. Satellite-borne x-ray
spectrometers are used to study x-ray emission lines from highly excited atoms in distant
astronomical sources. X-ray spectra are also used in air-pollution monitoring and in studies of
the abundance of various elements in rocks.

150
SUMMARY

The continuous x-ray spectrum is the result of the inverse photoelectric effect, with electron
kinetic energy being transformed into photon energy.

𝑚𝑒 4 1 1
Moseley’s law: 𝑓 = 8𝜖2 ℎ3 (𝑛2 − 𝑛2 ) (𝑧 − 1)2
𝑜 𝑓 𝑖

SELF ASSESMENT QUESTIONS


1. In an X-ray tube, electrons with energy 35 keV are incident on a cobalt (Z = 27) target.
Determine the cutoff wavelength for X-ray production.
(a) 1.4  10−11 m
(b) 2.8  10−11 m
(c) 3.6  10−11 m
(d) 1.8  10−11 m

2. Which electron energy will produce the lowest cutoff wavelength for X-ray production
from a nickel (Z = 28) surface?
(a) 25 keV
(b) 35 keV
(c) 45 keV
(d) 30 keV

3. Which one of the following statements concerning the cutoff wavelength typically exhibited
in X-ray spectra is true?
(a) The cutoff wavelength depends on the target material.
(b) The cutoff wavelength depends on the potential difference across the X-ray tube.
(c) The cutoff wavelength is independent of the energy of the incident electrons.
151
(d) The cutoff wavelength occurs because of the mutual shielding effects of K-shell
electrons.

4. Calculate the K X-ray wavelength for a gold atom (Z = 79).

(a) 5.13  10−10 m


(b) 2.00  10−11 m
(c) 2.47  10−13 m
(d) 3.60  10−11 m

5. What is the operating voltage of a medical X-ray machine that has a cut-off
wavelength of 2.20  10–11 m?
(a) 83 800 V
(b) 30 700 V
(c) 56 500 V
(d) 10 900 V

REFERENCES
Douglas C. Giancoli, (2009), 4th Edition. Physics for Scientists and Engineers with Modern
Physics, Pearson Educational Publishers.

Arthur Beiser (2003), Concepts of Modern Physics, 6th Edition, McGraw-Hill Publishers

Stephen T. Thornton and Andrew Rex (2013), Modern Physics for Scientists and Engineers, 4th
Edition, Charles Hartford Publishers

Should you require more explanation on this study session, please do not hesitate to contact your

e-tutor via the LMS.

152
Are you in need of General Help as regards your studies? Do not hesitate to contact
the DLI IAG Center by e-mail or phone on:

iag@dli.unilag.edu.ng
08033366677

STUDY SESSION 14: MOLECULAR SPECTROSCOPY

INTRODUCTION
The origin of spectral lines in molecular spectroscopy is the absorption, emission, and scattering
of a proton when the energy of a molecule changes. In molecules, there are energy states
corresponding to nuclei vibrations and rotations. As a result, the molecular spectrum contains
information on the molecular structure and the bond strength. They also provide a way of
determining a variety of molecular properties, like dipole and quadrupole moments and the
quantum numbers characterizing all molecular degrees of freedom. The molecular spectroscopy
is also important for astrophysical and environmental science, for investigation of chemical
reactions, for biology, and in many other areas of science and technology which needs detailed
investigation of properties of microscopic atomic and molecular objects.

LEARNING OUTCOMES
At the end of this study session, the student should be able to:

153
14.1 Highlight the basics concept of molecular spectroscopy
14.2 Calculate the bond lengths of diatomics from the value of their rotational constant.
14.3 Outline the selection rules for rotational and vibrational spectra

14.1 BASIC CONCEPT


Free atoms do not rotate or vibrate. For an oscillatory or a rotational motion of a pendulum, one
end has to be tied or fixed to some point. In molecules such a fixed point is the center of mass.
The atoms in a molecule are held together by chemical bonds. The rotational and vibrational
energies are usually much smaller than the energies required to break chemical bonds. The
rotational energies correspond to the microwave region of electromagnetic radiation (3x1010 to
3x1012 Hz; energy range around 10 to100 J/mol) and the vibrational energies are in the infrared
region (3x1012 to 3x1014 Hz; energy range around 10kJ/mol) of the electromagnetic radiation.
For rigid rotors (no vibration during rotation) and harmonic oscillators (wherein there are equal
displacements of atoms on either side of the center of mass) there are simple formulae
characterizing the molecular energy levels.
Additional energy levels become possible for molecules (but not for atoms) because the
molecule as a whole can rotate, and the atoms of the molecule can vibrate relative to each other.
The energy levels for both rotational and vibrational levels are quantized, and are generally
spaced much more closely (10-3 to 10-1 eV) than the electronic levels. Each atomic energy level
thus becomes a set of closely spaced levels corresponding to the vibrational and rotational
motions. Transitions from one level to another appear as many very closely spaced lines. In fact,
the lines are not always distinguishable, and these spectra are called band spectra. Each type of
molecule has its own characteristic spectrum, which can be used for identification and for
determination of structure.

In real life, molecules rotate and vibrate simultaneously and high speed rotations affect
vibrations and vice versa. However, in our introductory view of spectroscopy we will simplify
the picture as much as possible. We will first take up rotational spectroscopy of diatomic
molecules.

154
14.2 ROTATIONAL SPECTRA OF DIATOMICS

Fig.14.1 Diatomic molecule rotating about a vertical axis.

The two independent rotations of this molecule are with respect to the two axes which pass
though C and are perpendicular to the “bond length” r. The rotation with respect to the bond axis
is possible only for “classical” objects with large masses. For quantum objects, a “rotation” with
respect to the molecular axis does not correspond to any change in the molecule as the new
configuration is indistinguishable from the old one.

The centre of mass is defined by equating the moments on both segments of the molecular axis.
𝑚1 𝑟1 = 𝑚2 𝑟2 14.1
The moment of inertia is defined by
𝐼 = 𝑚1 𝑟1 2 + 𝑚2 𝑟2 2 14.2
𝐼 = 𝑚1 𝑟2 𝑟1 + 𝑚2 𝑟1 𝑟2
𝐼 = 𝑟1 𝑟2 (𝑚1 + 𝑚2 ) 14.3

155
Using equation 14.1, equation 14.3 becomes
𝑚1 𝑟1 = 𝑚2 𝑟2 = 𝑚2 (𝑟 − 𝑟1 ),
(𝑚1 + 𝑚2 )𝑟1 = 𝑚2 𝑟
Hence,
𝑚2 𝑟 𝑚1 𝑟
𝑟1 = (𝑚1 +𝑚2 )
and 𝑟2 = (𝑚1 +𝑚2 )
14.4

Substituting equation 14.4 into equation 14.3, we obtain

14.5

Where μ, is the reduced mass and is given by

14.6

The rotation of a diatomic is equivalent to a “rotation” of a mass μ at a distance of r from the


origin C. The kinetic energy of this rotational motion is K.E. = L2/2I where L is the angular
momentum, Iω where ω is the angular (rotational) velocity in radians/sec. The operator for L2 is
the same as the operator L2 for the angular momentum of hydrogen atom and the solutions of the
operator equations L2Υlm = l (l + 1) Υlm, where Υlm are the spherical harmonics.

The quantized rotational energy levels for this diatomic are

14.7
The energy differences between two rotational levels is usually expressed in cm-1. The wave
number corresponding to a given ΔE is given by
ν = ΔE /hc
The energy levels in cm-1 are therefore,


𝐸𝑗 = 𝐵𝐽(𝐽 + 1) 𝑤ℎ𝑒𝑟𝑒 𝐵 = 14.8
8𝜋2 𝐼𝑐

156
The rotational energy levels of a diatomic molecule are shown in figure 14.2

Figure 14.2: Rotational energy levels of a rigid diatomic molecule and the allowed
transitions.
The selection rule for a rotational transition is,
ΔJ=±1 14.9
In addition to this requirement, the molecule has to possess a dipole moment. As a dipolar
molecule rotates, the rotating dipole constitutes the transition dipole operator μ. Molecules such
as HCl and CO will show rotational spectra while H2, Cl2 and CO2 will not. The rotational
spectrum will appear as follows

Figure 14.3 Rotational spectrum of a rigid diatomic. Values of B are in cm-1. Typical values
of B in cm-1 are 1.92118 (CO), 10.593 (HCl), 20.956 (HF), 1H2 (60.864), 2H2 (30.442),
1.9987 (N2).
From the value of B obtained from the rotational spectra, moments of inertia I, of molecules, can
be calculated. From the value of I, bond length can be deduced.
157
Calculate the value of I and r of CO. B = 1.92118 cm-1.

h 6.625 × 10−34
I= =
8π2 Bc 8 × 3.1422 × 1.92118 × 3 × 1010
I = 1.456 × 10−46 𝑘𝑔𝑚2
Since the value of B is in cm-1, the velocity of light c is taken in cm/s. I = μr2. The atomic mass
of C ≡ 12.0000 amu, O ≡ 15.9994 amu. 1 amu = 1.6604 x 10-27 kg. The reduced mass of CO can
be calculated to be 1.13836 x 10-27 kg.
Hence,
𝐼 1.456 × 10−46
𝑟2 = = = 1.279 × 10−19
𝜇 1.138 × 10−27
𝑟 2 = 1.131𝐴̇.

14.3 VIBRATIONS AND ROTATIONS OF A DIATOMIC


You have noticed in your earlier studies that simple pendulums or stretched strings exhibit
simple harmonic motion about their equilibrium positions. Molecules also exhibit oscillatory
motions. A diatomic oscillates about its equilibrium geometry. The quantized vibration energies
Eυ of a harmonic oscillator are

Eυ = (u + ½) hν 14.10
v = 0,1,2,…………
The vibrational frequency ν is related to the force constant k through

158
1 k
v= √ 14.11
2π μ

The vibrational motion occurs under the action of a binding potential energy. The potential
energy (PE) curve for a harmonic oscillator is given in Figure 14.4

Figure 14.4. The potential energy of a harmonic oscillator V = k(r-ro)2.

On either sides of the equilibrium bond length ro, the PE rises as a symmetric quadratic function
(a parabola). The vibrational wave functions can be obtained by solving the Schrodinger
equation. The Hamiltonian operator (for energy) now consists of a kinetic energy term and a
potential energy term V as shown in Figure 14.4 and the solutions for energy, Ev have already
been given in equation 14.10. The selection rules for the harmonic oscillator are:
Δv = ± 1 14.12
We will see several equally spaced lines (spacing hν) corresponding to the transitions 0→1,
1→2, 2→3 and so on. The first transition will be the most intense as the state with v = 0 is the
most populated.
In actual diatomics, the potential is anharmonic. A good description of an anharmonic oscillator
is given by the Morse function.
P.E. = Deq [1 – exp [a(ro-r)]2 14.12
159
In Equation 14.12, Deq is the depth of the PE curve and ro is the bond length. A plot of the Morse
curve and the energy levels for the Morse potential are given in figure 14.5. The formula for the
energy levels of this anharmonic oscillator is
Ev/hc = ev = (v+ ½) ν - (v+ ½)2 ν xe, 14.13
Here xe, is called the anharmonicity constant whose value is near 0.01. It can be easily deduced
from the above formula that the vibrational energy levels for large υ start clustering together

Figure 14.5 Morse potential and the energy levels.


Note the difference between the dissociation energy Do and the depth Deq. All molecules have a
minimum of the zero point energy of hν/2 corresponding to the ν = 0 state. This is a consequence
of the uncertainty principle.
Often, one observes a combined vibrational rotational spectrum. A combined set of vibrational
and rotational energy levels of a diatomic is given by
Etotal = BJ (J + 1) + (v + ½ ) ν - xe (v+ ½)2 ν, cm-1 14.14
The energy level diagram and the spectrum corresponding to the diagram are shown in figure
14.6

160
Figure 14.6 The vibrational rotational spectrum

The selection rules are Δv = ± 1, ± 2,


Δ J = ± 1. Δ J = 1 corresponds to the R branch on the right at higher frequencies and
Δ J = J′′ – J′ = -1 corresponds to the P branch on the left. The dashed line Q for which
Δ J = 0, is not seen. The difference between R0 and P1 is 4B and the difference between adjacent
R lines and adjacent P lines is 2B.

SUMMARY
In this study session, we have discussed the following:
➢ When atoms combine to form molecules, the energy levels of the outer electrons are
altered because they now interact with each other.
➢ Additional energy levels also become possible because the atoms can vibrate with respect
to each other, and the molecule as a whole can rotate.
➢ The energy levels for both vibrational and rotational motion are quantized, and are very
close together.

161
➢ Each atomic energy level thus becomes a set of closely spaced levels corresponding to
the vibrational and rotational motions. Transitions from one level to another appear as
many very closely spaced lines. The resulting spectra are called band spectra.
➢ The quantized rotational energy levels are given by

where I is the moment of inertia of the molecule.


➢ The energy levels for vibrational motion are given by

where l is the classical natural frequency of vibration for the molecule. Transitions
between energy levels are subject to the selection rules ∆l = ±1 and ∆v = ±1.

SELF ASSESMENT QUESTIONS


(1) Show that the moment of inertia of a diatomic molecule rotating about its center of mass
can be written as

Where μ is the reduced mass and r is the distance between the atoms.

(2) Hydrogen molecule vibrations emit infrared radiation of wavelength around 2300 nm.
(a) What is the separation in energy between adjacent vibrational levels?
(b) What is the lowest vibrational energy state?

REFERENCES
Douglas C. Giancoli, (2009), 4th Edition. Physics for Scientists and Engineers with Modern
Physics, Pearson Educational Publishers.

162
Arthur Beiser (2003), Concepts of Modern Physics, 6th Edition, McGraw-Hill Publishers

Should you require more explanation on this study session, please do not hesitate to contact your

e-tutor via the LMS.

Are you in need of General Help as regards your studies? Do not hesitate to contact
the DLI IAG Center by e-mail or phone on:

iag@dli.unilag.edu.ng
08033366677
STUDY SESSION 15: ELECTRONIC
SPECTROSCOPY

INTRODUCTION
The Franck-Condon Principle originated in molecular spectroscopy in 1925 when J. Franck
proposed the idea that, when molecules absorb photons to undergo an electronic transition from
the ground state E0 to an excited state E1 The electronic transition occurs so rapidly that heavy
nuclei do not have time to rearrange to their new equilibrium positions. In this study session we
shall discuss the Born-Oppenheimer Principle and then take a step further in explaining the
Franck-cordon principle.

LEARNING OUTCOMES
At the end of this study session, the student should be able to
15.1 State the Born-Oppenhemimer Principle
15.2 State the Franck-Condon Principle
163
15.4 Explain the application of Franck-Condon principle in chemistry

15.1 BORN-OPPENHEIMER PRINCIPLE


The Born-Oppenheimer principle states that we can find the wavefunction for the electrons in
molecule without considering the motion of the nuclei. The reasoning for the Born-Oppenheimer
principle are:
(1) The nuclei are much, much heavier than the electrons.
(2) Thus the electrons respond much, much quicker to changes than the nuclei.
(3) Thus we can separate the electronic motion from the nuclear motion.
(4) Such a separation implies that we can write the total wavefunction of the system as the
product of an electronic wavefunction and a nuclear wavefunction.
(5) The nuclear wavefunction would the harmonic oscillator wavefunctions as we used in the
vibrational spectroscopy section.
Hence,
 molecule ( r, R ) = electronic ( r )  nuclear ( R ) 15.1

In the analysis of the electronic structure of a molecule, one does not need to account for the
motion of the nuclei. Sometimes the nuclear motion is important and creates what is called
vibronic coupling that is magnetic effect similar to spin-orbit coupling. We will not discuss
vibronic coupling further.

15.2 FRANCK-CONDON PRINCIPLE


Electronic transitions occur when electrons change their arrangement in an atom or molecule.
The Franck-Condon principle states that electronic transitions in molecules occur before nuclei
have time to adjust. One of the consequences of the Franck-Condon principle is that transitions
between electronic states is most likely to occur when the nuclei are “stationary”.

Nuclei are “stationary” where the vibrational wavefunction has the highest probability density
1.) For ground vibrational state, maximum probability occurs at equilibrium bond distance.

164
2.) For excited vibrational state, maximum probability occurs at turning points.

Turning points are positions where the motion of particle reverses itself. For oscillators, the
turning points are at the boundary of the potential. In other words, turning point is where kinetic
energy is zero and energy of particle is all potential energy.

FRANCK-CONDON PRINCIPLE IN CHEMISTRY


It is well established in inorganic chemistry that electron transfer processes must be preceded by
the reorganization of the solvation (also called hydration) shells surrounding reactants (Reynolds
and Lumry 1966). It was Libby (1952) who accounted for this phenomenon based on the
Franck-Condon principle, suggesting that, before the fast electron transfer can occur, the slower
nuclear rearrangements of water molecules in the hydration shells must take place, the proton
being 1,836 times as massive as the electron. This is schematically illustrated in Figure 15.1.

165
Figure 15.1 The Franck-Condon Principle in action in one of the simplest chemical
reactions known, i.e., the one-electron redox reaction of the iron ions.

The overall reaction involves the transfer of one electron from the ferrous ion, Fe+2, to the ferric
ion, Fe+3. Due to the charge difference, the hydration shell around the ferric ion is more compact
than the hydration shell around the ferrous ion. Despite this, there is a finite probability that the
two hydration shells assume similar sizes at some time points as depicted by the two identically
sized spheres partially overlapping in the upper portion of Figure 15.1. Such a transient,
metastable state is known as the Franck-Condon state or the transitions state, and it is only in this
state that one electron can be transferred from Fe+2 to Fe+3 resulting in the electron being on
either of the iron ions. That is, in the Franck-Condon state, the two iron ions are chemically
equivalent, within the limits set by the Heisenberg Uncertainty Principle (Reynolds and Lumry
1966). The Franck-Condon complex (i.e., the reaction system at the Franck-Condon state) can
now relax back to the reactant state or relax forward to the product state, depending on the sign
of the Gibbs free energy change, ΔG, accompanying the redox reaction. If ΔG given by
Equation 15.2 is negative, the reaction proceeds forward (from left to right), and if it is positive,
the reaction proceeds backward (from right to left).

ΔG = Gfinal – Ginitial = ΔG0 - RT log (Fe+2)/(Fe+3) . . . . . . . . . . . . . . (15.2)


where Gfinal and Ginitial are the Gibbs free energy levels of the final and initial states of the
reaction system, ΔG0 is the standard Gibbs free energy (i.e., ΔG at unit concentrations of the
reactants and products), R is the universal gas constant, T is the absolute temperature of the
reaction medium, and (Fe+2) and (Fe+3) are the concentrations of the ferrous and ferric ions.

SUMMARY
The Born-Oppenheimer principle states that we can find the wavefunction for the electrons in
molecule without considering the motion of the nuclei.
166
The Franck-Condon principle states that electronic transitions in molecules occur before nuclei
have time to adjust.

SELF ASSESSMENT QUESTIONS


(1) List five major reasons behind the Born-Oppenheimer principle.
(2) State one major consequence of Franck-Condon principle
(3) Briefly explain the Franck-Condon principle in chemistry

REFERENCES
Douglas C. Giancoli, (2009), 4th Edition. Physics for Scientists and Engineers with Modern
Physics, Pearson Educational Publishers.

Stephen T. Thornton and Andrew Rex (2013), Modern Physics for Scientists and Engineers, 4th
Edition, Charles Hartford Publishers

Should you require more explanation on this study session, please do not hesitate to contact your

e-tutor via the LMS.

Are you in need of General Help as regards your studies? Do not hesitate to contact
the DLI IAG Center by e-mail or phone on:

iag@dli.unilag.edu.ng
08033366677

167
SOLUTIONS
STUDY SESSION 1
(1) C
(2) A
(3) B
(4) B
(5) A
(6) D
(7) D
𝑓(1𝑜 ) 𝑐𝑜𝑡 2 (0.5𝑜 )
(8) The ratio is = = 4.00
𝑓(2𝑜 ) 𝑐𝑜𝑡 2 (1.0𝑜 )

𝑁(𝐴𝑢) 𝑛(𝐴𝑢)𝑡(79)2 792


(9) = = = 36.93
𝑁(𝐴𝑙) 𝑛(𝐴𝑙)𝑡(13)2 132

(10) See page 11

STUDY SESSION 2
(1) D
(2) D
(3) For one-electron ions other than hydrogen atoms, the expression for the wavenumber of
the lines in the electronic spectrum, must be modified because the energy of the levels
depends on the charge of the nucleus, Z.

Lines in the Brackett series have a lower state principal quantum number n1 = 4. The
lowest energy transition will be that to the next level so that n2 = 5. Thus, because for
Li2+, Z = 3

168
The wavelength of the transition is therefore

(4) For hydrogen atoms, the wavenumber of the lines in the electronic spectrum, is

Lines in the Paschen series have n1 = 3 so that, rearranging,

and so
n2 = 5

(5) The Rydberg constant for a hydrogenic atom or ion may be calculated from the effective
mass of the nucleus and electron pair and the fundamental constants using eqn 13.4b, so
that

with

Thus, since

then

169
(6)

(7)

𝑚𝑚𝑢𝑜𝑛 × 𝑚𝑝𝑟𝑜𝑡𝑜𝑛 (105.66)(938.27)


(8) 𝑅𝑒𝑑𝑢𝑐𝑒𝑑 𝑚𝑎𝑠𝑠; 𝜇 = = = 94.966𝑀𝑒𝑉/𝑐 2
𝑚𝑚𝑢𝑜𝑛 + 𝑚𝑝𝑟𝑜𝑡𝑜𝑛 105.66+938.27

170
ℏ 𝑚 ℏ 𝑚 0.511
(a) 𝑎0 = = (𝑚𝑘𝑒 2 ) = 𝑎0 = (0.0529𝑛𝑚) = 0.000285𝑛𝑚
𝜇𝑘𝑒 2 𝜇 𝜇 94.966

𝜇𝑘 2 𝑒 4 𝜇 𝑚𝑘 2 𝑒 4 94.966
(b) 𝐸0 = = ( )= (13.6𝑒𝑉) = 2.53 × 103 𝑒𝑉
𝑧ℏ2 𝑚 𝑧ℏ2 0.511

(c) Shortest wavelength Lyman line: n = ∞ to n =1, so its energy is Eo, then
ℎ𝑐
𝐸𝑜 =
𝜆
ℎ𝑐 1.24
𝜆= = = 0.49𝑛𝑚
𝐸𝑜 2.53

STUDY SESSION 3
(1) C
(2) A
(3) B
(4) A
(5) A
(6) D
(7) C
(8) See page 33

STUDY SESSION 4
(1) A
(2) D
(3) B
(4) A

171
(5)
Spontaneous emission Stimulated emission

1 Cannot be controlled from outside Is controlled from outside

2 Probabilistic or random process Energy transition takes place


between definite selected energy
levels
3 Emitted photons are random in
direction. Phase and state of Emitted photons are same in
polarization direction. Phase and state of
4 polarization
Not monochromatic
5 Are monochromatic
Not coherent
6 Are all coherent
Output is broad and less intense
7 Output narrow and highly intense.
In the output photons are not
multiplied. In the output photons are not
multiplied.

STUDY SESSION 5
(1) A
(2) B
(3) Ψ∗ Ψ = 𝐴2 𝑒𝑥𝑝[−𝑖(𝑘𝑥 − 𝑤𝑡) + 𝑖(𝑘𝑥 − 𝑤𝑡)] = 𝐴2
172
The condition for normalization becomes
𝑎 𝑎
∫ Ψ Ψdx = 𝐴 ∫ 𝑑𝑥 = 𝐴2 𝑎 = 1
∗ 2
0 0
1 1
𝐻𝑒𝑛𝑐𝑒, 𝐴 = 𝑎𝑛𝑑 Ψ = 𝑒𝑥𝑝[𝑖(𝑘𝑥 − 𝑤𝑡)]
√𝑎 √𝑎

(4)

(5) (a)

(b)

(6) See page 54

STUDY SESSION 6
173
(1) D
(2) D
(3) B
(4) E

(5) The probability of finding the particle between x1 and x2 when it is in the nth state is

𝑥2
2 𝑥2 𝑛𝜋𝑥
𝑃𝑋1𝑋2 = ∫ |Ψn |2 dx = ∫ 𝑠𝑖𝑛2 𝑑𝑥
𝑥1 𝐿 𝑥1 𝐿
𝑥 1 2𝑛𝜋𝑥 𝑥2
[ − 𝑠𝑖𝑛 ]
𝐿 2𝑛𝜋 𝐿 𝑥1
Here x1 = 0.45L and x2 = 0.55L. For the ground state, which corresponds to n = 1, we
have 𝑃𝑋1 𝑋2 = 0.198 = 19.8 percent
This is about twice the classical probability. For the first excited state which corresponds
to n = 2, we have 𝑃𝑋1𝑋2 = 0.0065 = 0.65 percent

STUDY SESSION 7
(1) D
(2) B
(3) D
(4) In general, if 𝐸 ≫ 𝑉𝑜
Then, 4𝐸(𝐸 − 𝑉𝑜 ) ≈ 4𝐸 2 . From binomial theorem (1+x)-1 ≈ 1 − 𝑥 for small x
𝑉𝑜 2 sin2 (𝑘2 𝐿 ) 𝑉𝑜 2 sin2 (𝑘2 𝐿 )
𝑅 ≈ 1 − [1 − ]=
4𝐸 2 4𝐸 2
2
𝑉𝑜 2 sin2 (𝑘2 𝐿 )
This can be written as 𝑅 ≈ [ ]
4𝐸 2

−1
𝑉𝑜 2 sin2 (𝑘2 𝐿 )
(5) 𝑇 = [1 + ]
4𝐸(𝐸−𝑉𝑜 )

(a) To obtain T = 1 we require sin (𝑘2 𝐿 ) = 0.


174
𝜋 𝜋ℎ 1 ℎ𝑐 1 1240
𝐿= = = = 0.220𝑛𝑚
𝑘2 √2𝑚(𝐸 − 𝑉𝑜 ) 2 √2𝑚𝑐 2 (𝐸 − 𝑉𝑜 ) 2 √2(511 × 103 )(7.8)

Any integer of this value will work


𝑛𝜋
(b) For maximum reflection sin2 (𝑘2 𝐿 ) = 1 𝑜𝑟 for any odd integer n. from
2𝑘2

the result above we see that the first maximum is with L equal to half the
value of L for the first minimum, or L = 0.110nm

(6)

(7) See page 68


(8) See page 71

STUDY SESSION 8
(1) C
(2) D
(3) J ranges from |𝐿 − 𝑆| to |𝐿 + 𝑆| or 2, 3, 4. Then in spectroscopic notation 2s+1LJ we have
three possibilities: 3F2, 3F3, 3F4. The ground state has the lowest J value or 3F2.
With n = 4, the full notation is 43F2.

175
(4) (a) 1S0: S = 0, L = 0, J = 0
(b) 2D5/2: S = 1/2, L = 2, J = 5/2
(c) 5F1: S = 2, L = 3, J = 1
(d) 3F4: S = 1, L = 3, J = 4

(5) See page 79


3 3 1 1 3
(6) For j = 𝑗 = state, gives mj = − 2 , − 2 , , .
2 2 2
1 1 1
For the 𝑗 = state, gives mj = − 2 , .
2 2

𝑒ℏ𝐵 (1.602 × 10−19 )(6.582×10−16 )(2.55)


(7) ∆𝐸 = = = 2.95 × 10−4 𝑒𝑉
𝑚 9.109×10−31

STUDY SESSION 9
(1) D
(2) B
(3) C
(4) B
(5) See page 90

STUDY SESSION 10
(1) B
(2) B
(3) C
(4) C
(5) B
(6) B
(7) D
(8) D

176
(9) For the 4f state n = 4 and l =3. The possible 𝑚𝑙 value are 0, ±1, ±2 and ±3 with 𝑚𝑠 =
±1/2= for each possible 𝑚𝑙 value. The degeneracy of the 4f state is then (with 2 spin
states per 𝑚𝑙 ) equal to 2(7) = 14.

(10) For the 5d state n = 5 and l =2. The possible 𝑚𝑙 value are 0, ±1, ±2 and ±3 with
𝑚𝑠 = ±1/2. For each possible 𝑚𝑙 value. The degeneracy of the 5d state is then (with 2
spin states per 𝑚𝑙 ) equal to 2(5) = 10.

STUDY SESSION 11
(1) D
(2) C
(3) D
(4) B
(5) B
(6) D
(7) A
(8) C
(9) D
(10) See page 120

STUDY SESSION 12
(1) B
(2) D
(3) B
(4) A
(5) C
(6) C
(7) See page 118
(8) See page 121
177
STUDY SESSION 13
(1) C
(2) C
(3) B
(4) B
(5) C

STUDY SESSION 14
(1) The moment of inertia of a single particle of mass m a distance r from the rotation axis is
I = mr2 For our diatomic molecule

(2) The energy separation between adjacent vibrational levels is ∆Evib = h f = hc/λ. The lowest
energy has v = 0.
178
ℎ𝑐 (6.63 × 10−34 )(3 × 108 )
(a) ∆𝐸𝑣𝑖𝑏 = ℎ𝑓 = = = 0.54𝑒𝑉
𝜆 (2300 × 10−9 )(1.6 × 10−19 )

where the denominator includes the conversion factor from joules to eV


1 1
(b) ∆𝐸𝑣𝑖𝑏 = (𝑣 + 2) ℎ𝑓 = ℎ𝑓 = 0.27𝑒𝑉
2

STUDY SESSION 15
(1) See page 147
(2) See page 148
(3) See page 151

179

You might also like